You are on page 1of 65

PART IV

Pathogenic Fungi
J. And re w Alsp a ug h

Fungi—Basic Concepts C HA P T ER 4 2
Pathogenesis and Diagnosis of Fungal Infections C HA P T ER 4 3
Antifungal Agents and Resistance C HA P T ER 4 4
The Superficial and Subcutaneous Fungi: Dermatophytes, Malassezia, Sporothrix, and
Pigmented Molds C HA P T ER 4 5
The Opportunistic Fungi: Candida, Aspergillus, the Zygomycetes, and Pneumocystis C HA P T ER 4 6
The Systemic Fungal Pathogens: Cryptococcus, Histoplasma, Blastomyces, Coccidioides,
Paracoccidioides C HA P T ER 4 7
This page intentionally left blank
42
Fungi—Basic Concepts

Overview
The fungal kingdom encompasses a diverse and rich group of organisms ranging from
microscopic yeasts to mushrooms. Most fungi are free-living in nature where they function
as decomposers in the energy cycle. Of the more than 90 000 known fungal species, fewer
than 200 have been reported to produce disease in humans. Once considered clinical
rarities, human fungal infections are becoming increasingly common, especially among
immunocompromised patients. Therefore, it is important to understand the unique clinical
and microbiological features of these diseases.

MYCOLOGY
Fungi are eukaryotes with a higher level of biologic complexity than bacteria. They are
spore-bearing, reproducing both sexually and asexually. Fungi may be unicellular or may
differentiate and become multicellular by the development of long, branching filaments.
They lack the chlorophyll of plants, therefore needing to acquire nutrients from the exter-
✺ Cell organization is eukaryotic
nal environment. The diseases caused by fungi are called mycoses. These infections vary
greatly in their manifestations but tend to present with subacute or chronic features, often
relapsing over time. Acute disease, such as that produced by many viruses and bacteria, is
uncommon with fungal infections.

STRUCTURE
The fungal cell has many typical eukaryotic features, including a nucleus with a nucleo-
lus, nuclear membrane, and linear chromosomes (Figure 42–1). The cytoplasm contains a
cytoskeleton with actin microfilaments and tubulin-containing microtubules. Ribosomes
and organelles, such as mitochondria, endoplasmic reticulum, and the Golgi apparatus, are
also present. Fungal cells have a rigid cell wall external to the cytoplasmic membrane, which Presence of a nucleus, mitochon-
differs in its chemical composition from the cell walls of bacteria and plants. In addition to dria, and endoplasmic reticulum
the cell wall, another important difference from mammalian cells is the sterol makeup of the
cytoplasmic membrane. In mammalian cells, the dominant membrane sterol is cholesterol; ✺ Ergosterol, not cholesterol,
in fungi, it is ergosterol. Fungi are usually haploid in their DNA content, although diploid makes up cell membrane
nuclei are formed through nuclear fusion in the process of sexual reproduction. Interest-
ingly, the generation of polyploid/aneuploid nuclei is a strategy used by some fungi to gen-
erate genetic diversity as a response to cell stress, such as antifungal therapy.
The chemical structure of the cell wall in fungi is markedly different from that of bacterial
cells in that it does not contain peptidoglycan, glycerol, teichoic acids, or lipopolysaccha-
ride. In their place are complex polysaccharides such as mannans, glucans, and chitins in
close association with each other and with structural proteins (Figure 42–2). Mannoproteins
are composed of mannose-based polymers (mannan) found on the cell surface and in the

739
740 PART IV PAT H O G E N I C F U N G I

Cell wall
Nucleus
Cytoplasm
Polar bud
scar
Polar-
bud
scar
Chromosome

Plasma
membrane

FIGURE 42–1. A yeast cell showing


the cell wall and internal structures Mitochondrion Nuclear
of the fungal eukaryotic cell plan. envelope
(Reproduced with permission from Golgi apparatus
Willey JM: Prescott, Harley, & Klein’s
Microbiology, 7th edition. McGraw-Hill,
2008.) Plasma membrane

structural matrix of the cell wall, where they are linked to various proteins. Mannoproteins
are very important since antibodies are readily developed against these molecules on the
cell surface. The potential variations in the composition and linkages of the mannan side
chains allow fungi to generate a complex and adaptable cell surface to avoid easy immune
✺ Cell wall mannan linked to
detection by an infected host. The identification of different antibodies directed against
surface proteins
specific mannoproteins also allows laboratories to “serologically” distinguish between indi-
vidual strains within a fungal species. The alpha- and beta-glucans are polymers of glucose
✺ Chitin and glucans give rigidity
found abundantly throughout the cell wall. Additionally, chitin, composed of long chains
to cell wall
of N-acetylglucosamine, provides rigid structural support to the fungal cell in a manner
analogous to the chitin in crab shells or cellulose in plants. In addition to their structural
roles, these cell wall carbohydrates serve complex cell functions, often simultaneously acti-
vating and inhibiting various arms of the host immune response.

Fibrillar proteins

Mannoprotein

Glucan Ergosterol

Chitin
FIGURE 42–2. The fungal cell wall.
The overlapping mannan, glucan,
chitin, and protein elements are shown.
Proteins complexed with the mannan Cytoplasmic
membrane
(mannoproteins) extend beyond the
cell wall.
CHAPTER 42 F U N G I — B A S I C CO N C E P T S 741741

METABOLISM
A major difference between fungi and plants is that fungi lack chloroplasts and photosyn-
thetic energy-producing mechanisms. Therefore, fungi must acquire nutrients from exog-
enous sources. Metabolic diversity among fungi is great, but most are able to grow with very Heterotrophic metabolism uses
simple carbon and nitrogen sources. In nature, nutrients for free-living fungi are derived available organic matter
from decaying organic matter. Most fungi are strict aerobes, although some can grow under
anaerobic conditions.

REPRODUCTION
Fungi may reproduce by either asexual or sexual process. The asexual form is called the
anamorph, and its reproductive elements are termed conidia. The sexual form is called the
teleomorph, and its reproductive structures are called spores (eg, ascospores, zygospores,
basidiospores). Asexual reproduction involves mitotic division of the haploid nucleus and ✺ Asexual reproduction forms
is associated with production by budding, spore-like conidia or, alternatively by the separa- conidia by mitosis
tion of hyphal elements. In sexual reproduction, the haploid nuclei of donor and recipient
cells fuse to form a diploid nucleus, which then divides by classic meiosis. Some of the four ✺ Meiosis forms sexual spores in
resulting haploid nuclei may be genetic recombinants and may undergo further division specialized structures
by mitosis. Highly complex specialized structures may be involved. Detailed study of the
mating process in fungal species, such as the baker’s yeast Saccharomyces cerevisiae and the
red bread mold Neurospora crassa, has been important in gaining an understanding of basic
cellular genetic mechanisms.

FUNGAL MORPHOLOGY AND GROWTH


The size of fungi varies immensely. A single cell without transverse septa may range from
bacterial size (2-4 μm) to a macroscopically visible structure. The morphologic forms of
growth vary from colonies superficially resembling those of bacteria to some of the most Vary from bacterial size to multi-
complex, multicellular, colorful, and beautiful structures seen in nature. Mushrooms are cellular mushrooms
an example and can be regarded as a complex organization of cells showing structural
differentiation.
Mycology, the science devoted to the study of fungi, has various terms to describe the
morphologic components that comprise these structures. The terms and concepts that
must be mastered can be limited by considering only the fungi of medical importance and
accepting some simplification.

YEASTS AND MOLDS


Fungi that cause human infections can be broadly divided based on their morphological
forms. Yeasts are fungi that primarily grow in a round cellular form. Molds are fungi that
primarily grow as filamentous, tube-like structures called hyphae (Figure 42–3A and B).
Although it is useful to consider this basic distinction based on cell shape, it is important
to remember that some fungi can transition between yeast-like and hyphal morpholo- Yeasts produce blastoconidia by
gies. Often, this plasticity of shape is directly related to pathogenesis since different forms budding
may be better suited for different microenvironments. The yeasts tend to have the sim-
plest cellular forms, reproducing by a process of asexual budding, constriction, and cell
separation similar to many bacteria. The newly formed daughter cell is often called a
blastoconidium.
Fungi may also grow through the development of hyphae (singular, hypha), which are
tube-like extensions of the cell with thick, parallel walls. As the hyphae extend, they form
an intertwined mass called a mycelium. Most molds form hyphal septa (singular, septum),
which are cross-walls perpendicular to the cell walls, dividing the hypha into subunit cells
(Figure 42–4). The structure of these septa varies among species and may contain pores
and incomplete walls that allow movement of nutrients, organelles, and nuclei between
adjacent cells. Some species, including some human pathogens, form septae that are very
distant from each other. Because their microscopic appearance therefore suggests a single,
742 PART IV PAT H O G E N I C F U N G I

FIGURE 42–3. Yeast and mold


forms of fungal growth. A. This
oval yeast cell is budding to form
a blastoconidium. Scars from prior
separations of other blastoconidia can
be seen on other parts of the cell. B.
The mold form is highly variable. Here
tubular stalks called conidiophores
arising from hyphae (not seen) bear a
“Medusa head” crop of reproductive
conidia. (Reproduced with permission
from Willey JM: Prescott, Harley, & Klein’s
Microbiology, 7th edition. McGraw-Hill, A Saccharomyces cerevisiae:
2008.) budding division B

continuous hyphal cell, these particular species are often called “aseptate” molds. In both
septate and aseptate hyphae, multiple nuclei are often present in each cell.
A portion of the mycelium (vegetative mycelium) usually grows into the medium or
organic substrate (eg, soil) and functions like the roots of plants as a collector of nutrients

Nucleus

Nucleus Septum
B

Pores
FIGURE 42–4. Hyphae. A.
Nonseptate hyphae with multiple
nuclei. B. Septate hyphae divide
nuclei into separate cells. C. Electron
micrograph of septum with a single
pore. D. Multipore septum structure.
(Reproduced with permission from
Willey JM: Prescott, Harley, & Klein’s
Microbiology, 7th edition. McGraw-Hill,
D
2008.)
CHAPTER 42 F U N G I — B A S I C CO N C E P T S 743743

and moisture. The more visible surface growth may assume a fluffy character as the myce- ✺ Molds produce septate or
lium becomes aerial. The hyphal walls are rigid in order to support this extensive, intertwin- aseptate hyphae
ing network. The aerial hyphae bear the reproductive structures of this class of fungi. These
sexual structures are often unique to each species, allowing microbiology laboratories to ✺ Vegetative mycelium acts as a
distinguish among molds based on their morphological features. Some fungi also form root
pseudohyphae, which are actually elongated yeast cells growing end-to-end. Therefore,
pseudohyphae are distinguished from true hyphae by having recurring bud-like constric- ✺ Aerial mycelium bears
tions and less rigid cell walls. reproductive conidia or
Exogenously formed conidia may develop directly from the hyphae or on a special spores—the basis for species
stalk-like structure, the conidiophore (Figure 42–3B). Occasionally, terms such as identification
macroconidia and microconidia are used to indicate the size and complexity of these
conidia. Conidia that develop within the hyphae are called either chlamydoconidia or ✺ Pseudohyphae are elongated
arthroconidia. Chlamydoconidia become larger than the hypha itself; they are round, yeast-like cells
thick-walled structures that may be borne on the terminal end of the hypha or along
its course. Arthroconidia conform more to the shape and size of the hyphal units but Conidia and conidiophore
are thickened or otherwise differentiated. Arthroconidia may form a series of delicately arrangements determine names
attached conidia that break off and disseminate when disturbed. Some of the asexual repro-
ductive forms are illustrated in Figure 42–5A–D. The most common sexual spore is termed Ascospores are borne in
an ascospore. Four or eight ascospores may be found in a sac-like structure, the ascus. ascus sac

Terminal chlamydoconidia
Arthroconidia

Fragmenting hypha

Chlamydoconidia within
a hypha

A B

Sporangiospores
Conidioconidia

Sporangium FIGURE 42–5. Asexual mold forms.


A. Arthroconidia develop within the
hyphae and eventually break off. B.
Chlamydoconidia are larger than the
Sporangiophore
hyphae and develop with the cell or
terminally. C. Sporangioconidia are
borne terminally in a sporangium sac.
Conidiophore
D. Simple conidia arise directly from
a conidiophore. (Reproduced with
permission from Willey JM: Prescott,
Harley, & Klein’s Microbiology, 7th edition.
C D McGraw-Hill, 2008.)
744 PART IV PAT H O G E N I C F U N G I

DIMORPHISM
Although many fungi tend to grow as either yeasts or molds, some species can transition
between morphological forms depending on environmental conditions. These species are
known as dimorphic fungi. Many fungi, including the most common human fungal patho-
gen Candida albicans, display a striking ability to modify their cellular shape and structure
in order to adapt to new environments. These morphological transitions are often very
important for the pathogenesis of human infections.
A distinct group of human pathogens are the thermally dimorphic fungi, shifting from
yeast-like to hyphal growth based on temperature. These fungal species tend to grow in the
mold form in their environmental reservoir as well as when incubated in culture at ambient
✺ Growth in yeast or mold form temperatures. However, they convert to a yeast-like growth form in the mammalian host
or when incubated in culture at 37°C. For most, it is possible to manipulate the culture
✺ Temperature triggers shift conditions to demonstrate both yeast and mold phases in vitro. Yeast phase growth requires
between phases conditions similar to those of the physiologic in vivo environment, such as 35°C to 37°C
incubation and enriched medium. Mold growth requires minimal nutrients and ambient
temperatures. Importantly, the conidia produced in the mold phase may be infectious and
serve to disseminate the fungus during growth in the environment.
The morphologic and physiologic events associated with the temperature-dependent
conversion between the mold and yeast phases have been studied in the human pathogen
Histoplasma capsulatum. They are understandably complex, given the dramatic change of
milieu encountered by the fungus when its mold conidia float from their soil habitat to the
pulmonary alveoli. Conversion to the yeast phase is then triggered by the host temperature
Shift from mold to yeast begins (37°C) and other conditions associated with the host microenvironment (eg, iron limita-
with heat shock response tion, pH changes, elevated levels of CO2). In vitro studies show that the earliest events in
this morphological shift involve induction of the heat shock response and uncoupling of
✺ Metabolic shift is toward oxidative phosphorylation. These early cellular events are followed by a shutdown of RNA
sulfhydryl compounds in yeast synthesis, protein synthesis, and respiratory metabolism. The cells then pass through a met-
form abolically inactive state, emerging with enhanced enzymatic capacities involving sulfhydryl
compounds (eg, cysteine, cystine) that are exclusive to the yeast stage. As yeast growth
develops, there is recovery of mitochondrial activity and synthetic capacity, but a new
constellation of oxidases, polymerases, proteins, cell wall glucans, and other compounds
are present, likely favoring host parasitism. In all, more than 500 genes are differentially
expressed in the mold and yeast phases.
Dimorphism in fungi is reversible, a feature that distinguishes it from developmental
processes such as embryogenesis seen in higher eukaryotes. The importance of the dimor-
✺ Dimorphism is reversible and phism in fungal virulence has been demonstrated in several fungi, including C albicans and
linked to virulence H capsulatum. Strains that are locked in one growth phase are markedly reduced in their
ability to produce disease and persist in the host.

CLASSIFICATION
✺ Taxonomy is based on sexual Fungal classification has historically relied upon observable cellular characteristics such
spores and septation of hyphae as the septation of hyphae and the appearance of the sexual structures. However, DNA
sequence-based classification methods are becoming more common, allowing fungi to be
grouped by genetic relatedness. Molecular classification techniques have also demonstrated
rRNA genes are used for
that several microbial species are actually fungi despite having few fungal growth charac-
classification
teristics (eg, Pneumocystis species and Microsporidia).
Fungi have historically been organized into five phyla: Ascomycota, Basidiomycota,
Zygomycota, Chytridiomycota, and Glomeromycota. The medically important genera fall
mostly within the Ascomycota, with a few in Basidiomycota, and Zygomycota, as shown in
Table 42–1.
The grouping of medically important fungi used in the following chapters is based on the
types of tissues they parasitize and the diseases they produce, rather than on the principles
of basic mycologic taxonomy. The superficial fungi, such as the dermatophytes, cause indo-
lent lesions of the skin and its appendages, commonly known as ringworm and athlete’s
foot, without typically spreading to deeper tissues. The subcutaneous pathogens charac-
teristically cause infection through the skin, followed by subcutaneous or lymphatic spread.
CHAPTER 42 F U N G I — B A S I C CO N C E P T S 745745

TABLE 42–1 Classification of Medically Important Fungi


GENUS TYPICAL GROWTH SEPTATIONa PHYLUM
Superficial Fungi
Epidermophyton Mold + Ascomycota
Microsporum Mold + Ascomycota
Trichophyton Mold + Ascomycota
Subcutaneous Fungi
Sporothrix Dimorphic + Ascomycota
Opportunistic Fungi
Aspergillus Mold + Ascomycota
Candida Dimorphic + Ascomycota
Mucor Mold – Zygomycota
Pneumocystis Cystsb Ascomycota
Rhizopus Mold – Zygomycota
Systemic Fungi
Blastomyces Dimorphic + Ascomycota
Coccidioides Dimorphic + Ascomycota
Cryptococcus Yeast Basidiomycota
Histoplasma Dimorphic + Ascomycota

For those that form hyphae.


a

Tissue forms but does not grow in culture.


b

The opportunistic fungi are those found in the environment or in the resident flora that
produce disease primarily in immunocompromised hosts. The systemic pathogens are
Medical grouping organized by
the most virulent fungi and may cause serious and progressive systemic disease in previ-
biologic behavior in humans
ously healthy persons. They are not found in the human microbiota. Although their major
potential is to produce deep-seated visceral infections and systemic spread (systemic myco-
Systemic fungi infect previously
ses), they may also produce superficial infections as part of their disease spectrum or as the
healthy persons
initiating event. As with all clinical classifications, overlaps and exceptions occur. In the
end, the interplay between host and microorganism defines the disease.
This page intentionally left blank
43
Pathogenesis and
Diagnosis of Fungal
Infections
Overview
We all have regular contact with fungi. They are so widely distributed in our environment
that thousands of fungal spores are inhaled or ingested every day. Some species are so
well adapted to humans that they are common members of the microbiota. Despite
this ubiquity, clinically apparent systemic fungal infections are uncommon, even among
persons living within the geographic habitat of the more pathogenic species. However,
progressive systemic fungal infections pose some of the most difficult diagnostic and
therapeutic problems in infectious disease, particularly among immunocompromised
patients to whom they are a major threat. The purpose of this chapter is to provide an
overview of the pathogenesis and immunology of fungal infections. Details relating to
specific fungi are provided in Chapters 45 to 47.

GENERAL ASPECTS OF FUNGAL DISEASE EPIDEMIOLOGY


Fungal infections are most often acquired from the external environment. One common
mechanism of infection is by the inhalation of infectious conidia generated from environ- Environmental conidia are
mental molds. Some of these molds are ubiquitous, whereas others are restricted to specific inhaled
endemic areas and geographic regions whose climate favors their growth. Many fungi pro-
duce disease only after they are accidentally injected past the skin/mucosal barrier, espe- ✺ Certain fungi are endemic to
cially in immunocompromised patients. Other pathogenic fungi have more sophisticated specific geographic regions
means of tissue penetration and invasion. In the case of systemic candidiasis, infection can
result from systemic invasion by a fungal species that is typically an endogenous member of Endogenous yeasts may invade
the resident microbiota, such as Candida albicans (Figure 43–1).

PATHOGENESIS
Compared with bacterial, viral, and parasitic disease, less is known about the pathogenic
mechanisms and virulence factors involved in fungal infections. Analogies to bacterial dis-
eases come the closest because of similarities in microbial adherence to mucosal surfaces, Fungal pathogenesis is similar to
invasion into deeper tissue layers, production of extracellular compounds, and interaction bacteria
with phagocytes (Figure 43–2). In general, the principles discussed in Chapter 22 also apply
to fungal infections. Most fungi are opportunists, causing serious disease only in individu- Most fungi are opportunists
als with impaired host defense systems. Only a few fungi are able to cause disease in previ-
ously healthy persons.

747
748 PART IV PAT H O G E N I C F U N G I

Localized Systemic
disease disease

Thrush

Bone

FIGURE 43–1. Fungi system view.


Localized disease (left) is caused by Meningitis
local trauma or the superficial invasion
of flora resident on the oropharyngeal
(thrush), gastrointestinal, or vaginal
mucosa. Systemic fungal disease (right)
most often begins with inhalation of
conidia followed by dissemination to
other sites. Vaginitis

● Adherence
Several fungal species, particularly the yeasts, are able to colonize the mucosal surfaces of
the gastrointestinal and female genital tracts. It has been shown experimentally that the
ability to adhere to buccal or vaginal epithelial cells is associated with colonization and
virulence. Within the genus Candida, the species that best adhere to epithelial cells are
Adherence is mediated by fungal those most frequently isolated from clinical infections. Adherence usually requires a sur-
adhesins and host cell receptors face adhesin on the fungus and a receptor on the epithelial cell. In the case of C albicans,
mannoprotein components extending from the cell wall have been implicated as specific
adhesins, interacting with host fibronectin and other components of the extracellular
matrix. Other fungal/host binding mediators have been identified, and this process can

Cytokines Bound
FIGURE 43–2. Immunity to fungal cytokines
infections. A. Pathogenic fungi are
able to survive and multiply slowly in
nonactivated macrophages. B. When
macrophages are activated by cytokines
from T-cells, the growth is restricted and T cell
the fungi digested. A B
CHAPTER 43 PAT H O G E N E S I S A N D D I A G N O S I S O F F U N G A L I N F E C T I O N S 749749

help to explain why certain tissues are targeted by specific fungal pathogens. For example,
the neuropathogen Cryptococcus neoformans displays a unique interaction with proteins on
the endothelium of the brain microvasculature, perhaps explaining how this species specifi-
cally invades the central nervous system.

● Invasion
Passing an initial surface barrier—skin, mucous membrane, or respiratory epithelium—
is an important step for most successful pathogens. Some fungi are introduced through Traumatic injection is linked to
mechanical breaks. For example, Sporothrix schenckii infection typically follows a thorn trauma
prick trauma to the skin. Fungi that initially infect the lung must produce conidia small
enough to be inhaled past the upper airway defenses. For example, arthroconidia of Small conidia may pass airway
Coccidioides immitis (2-6 μm) can remain suspended in air for a considerable period of defenses
time and can reach the terminal bronchioles to initiate pulmonary coccidioidomycosis.
Triggered by temperature and possibly other cues, dimorphic fungi from the environ-
ment undergo a metabolic shift similar to the heat shock response, completely changing
their morphology to a more invasive form. Invasion directly across mucosal barriers by
the endogenous yeast C albicans is similarly associated with a morphologic change, the
formation of hyphae. For this species, the ability to transition between yeast-like and hyphal Invasion across mucosal barriers
forms allows it to effectively penetrate tissue, form adherent biofilms, and disseminate to may involve enzymes
distant sites. Extracellular enzymes (eg, proteases, elastases) are associated with the advanc-
ing edge of the hyphal form of Candida species, as well as with the invasive forms of many
of the dimorphic and other pathogenic fungi.

● Injury
There are many mechanisms of tissue injury during fungal infection. Although many fungi
produce secondary metabolites and mycotoxins in the environment, most of these extra-
cellular toxins do not appear to be directly related to pathogenesis in human infections.
No classic exotoxins are pro-
Cell surface components contributing to host damage, analogous to the endotoxin of
duced in vivo
gram-negative bacteria, are lacking in most fungi. Moreover, only the most immunocom-
promised patients appear to have extensive injury due to direct fungal destruction of the
✺ Injury is due to inflammatory
surrounding tissue, such as neutropenic patients with invasive mold infections. In contrast,
and immunologic responses
the injury experienced by the host during most fungal infections seems to be due primar-
ily to the immune response against the infecting microorganism. As the immune system
attempts to clear the fungal pathogens, there is some degree of collateral damage to the host.

IMMUNITY

● Innate Immunity
Healthy persons have effective innate immunity to most fungal infections, especially the
opportunistic molds. This resistance is mediated by the professional phagocytes (neutro-
phils, macrophages, and dendritic cells), the complement system, and pattern recognition
receptors. Important receptors recognizing fungal elements include a lectin-like structure
on phagocytes (dectin-1) that binds glucan, and toll-like receptors (TLR2, TLR4). In most
instances, neutrophils and alveolar macrophages are able to kill the conidia of fungi if they
reach the tissues.
Fungal species that cause human infections have developed strategies to avoid immune
recognition and to thwart various aspects of immune-mediated clearance. The polysac-
charide capsule of C neoformans shields immunogenic epitopes on the cell surface from
being sensed by pattern recognition receptors and complement proteins. Moreover, capsule
material secreted by the cryptococcal cell specifically inhibits the function of many immune Most fungi are readily killed by
cells. Similarly, Candida albicans is able to bind complement components in a way that neutrophils
interferes with phagocytosis. As the thermally dimorphic fungi convert to the pathogenic
yeast-like state, they too become more resistant to killing by phagocytes than fungal oppor-
tunists because of changes in surface structures subject to pattern recognition.
In addition to preventing immune recognition, many fungal pathogens are also able
to survive once sensed and engulfed by immune cells. The yeast-to-hyphal transition by
750 PART IV PAT H O G E N I C F U N G I

C albicans favors its escape from phagocytic immune cells. As the hyphae of the thermally
dimorphic fungus C immitis convert to the spherule (tissue) phase, they also become resis-
✺ Pathogenic fungi resist tant to phagocytic killing because of their size and surface characteristics. Some fungi pro-
phagocytic killing duce substances such as melanin, which interfere with oxidative killing by phagocytes. The
yeast forms of Histoplasma capsulatum and C neoformans are adapted to live and multiply
within macrophages by interfering with lysosomal killing mechanisms.

● Adaptive Immune Response


A recurrent theme with fungal infections is the importance of an intact immune response
in preventing infection and progression of disease. Most fungi are incapable of producing
✺ T-cell–mediated responses of
even a mild infection in immunocompetent individuals. The small number of species that
primary importance
are able to cause clinically apparent infection are usually cleared from the host, most often
through a combination of the innate activity of neutrophils and through the development
Progressive fungal diseases occur
of an adaptive, TH1-mediated immune response. Progressive, debilitating, or life-threatening
in the immunocompromised
fungal infections are commonly associated with depressed or absent cellular immune
responses.

● Humoral Immunity
Antifungal antibodies can be detected at some time during the course of almost all fungal
infections, but the appearance of antibodies does not necessarily correlate with resistance.
In coccidioidomycosis, for example, high titers of C immitis-specific antibodies are associ-
Opsonizing antibody is effective ated with dissemination and a worsening clinical course; antibody titers decrease as the
in some yeast infections infection is cured. In contrast, antibodies directed against the C neoformans capsule may
actually contribute to the cell-mediated clearance of this encapsulated yeast from the site
of infection. Antibody may also play a role in control of C albicans infections by enhancing
fungus–phagocyte interactions.

● Cellular Immunity
Considerable clinical and experimental evidence points toward the importance of cellular
Systemic disease associated with immunity in the resolution of fungal infections. Most patients with invasive mycoses have
deficiencies in neutrophils and neutropenia, defects in neutrophil function, or depressed TH1 immune responses. These
T-cell–mediated immunity can result from factors such as steroid treatment, leukemia/lymphoma, transplantation, and
AIDS.
A basic schema for fungal-host interactions is illustrated in Figure 43–2. When hyphae
or yeast cells of the fungus reach deep tissue sites, they are either killed by neutrophils or
resist destruction by one of the antiphagocytic mechanisms described earlier. Surviving
cells continue to grow slowly within the host in their tissue-adapted fungal forms (spher-
Fungi that escape neutrophils
ules for C immitis, hyphae for A fumigatus, intracellular yeasts for C neoformans and
grow slowly in macrophages
H capsulatum). The growth of these invasive forms may be slowed or killed by phagocytes
such as neutrophils and macrophages. In healthy persons, the extent of infection is mini-
mal, and any symptoms are caused by the inflammatory response. Fungal persistence and
spread is most common in people with defective immunity.
Everything awaits the specific adaptive immune response to the invader. In fungal infec-
tions, antigen presenting cells such as dendritic cells and macrophages help to activate
Immune defects lead to progres-
adaptive immune response, including antifungal antibody production and TH1-mediated
sive disease
immunity. Defects that disturb this cycle lead to progressive disease. To the extent that they
are known, the specifics of these reactions are discussed in the following chapters.

LABORATORY DIAGNOSIS

● Direct Examination
Fungi can often be identified by directly observing their distinctive morphologic features on
direct microscopic examination of infected pus, fluids, or tissues. The simplest method is to
✺ KOH digests tissue, but not mix a clinical specimen, such as skin scrapings, with a 10% solution of potassium hydroxide
fungal wall (KOH) on a microscope slide under a coverslip. The strong alkali digests the tissue elements
(epithelial cells, leukocytes, debris), but not the rigid cell walls of either yeasts or molds.
CHAPTER 43 PAT H O G E N E S I S A N D D I A G N O S I S O F F U N G A L I N F E C T I O N S 751751

FIGURE 43–3. KOH (potassium


hydroxide) preparation. Scalp
scrapings from a suspected ringworm
lesion have been mixed with 10% KOH
and viewed under low power. The skin
has been dissolved, revealing tubular
branching hyphae.

After digestion of the material, the fungi can be observed under the light microscope with Some yeasts are gram positive
or without staining (Figure 43–3). Direct examinations can be aided by the use of calcofluor
white, a dye that binds to polysaccharides in cellulose and chitin. Under ultraviolet light, Calcofluor white enhances
calcofluor white fluoresces, enhancing detection of fungi in fluids or tissue sections. A few detection by staining fungal
yeasts including C albicans can be visualized using the Gram stain (gram positive). chitin
Histopathologic examination of tissue biopsy specimens is widely used to diagnose
fungal infections and shows the relation of the organism to tissue elements and responses
(blood vessels, phagocytes, granulomatous reactions). Most fungi can be seen in sections Fungi often visible in
stained with the basic hematoxylin and eosin (H&E) method used in histology laboratories H&E preparations
(Figure 43–4). Specialized staining procedures such as silver impregnation methods are fre-
quently used because they stain almost all fungi strongly, but only a few tissue components Silver stains enhance detection
(Figure 43–5). The pathologist should be alerted to the possibility of fungal infection when
tissues are submitted, because fungal-specific stains are used routinely.

● Culture
Fungi can be grown by methods similar to those used to isolate bacteria. The growth of
many fungal species occurs readily on enriched bacteriologic media commonly used in Growth in culture is simple but
clinical laboratories (eg, blood agar and chocolate agar). Many fungal cultures, however, slow
require days to weeks of incubation for initial growth; bacteria present in the specimen
grow more rapidly and may interfere with isolation of a slow-growing fungus. Therefore, Selective media allow isolation in
the culture procedures of diagnostic mycology are designed to favor the growth of fungi the presence of bacteria
over bacteria and to allow incubation to continue for a sufficient time to isolate slower
growing strains.

FIGURE 43–4. Disseminated


candidiasis. Candida albicans (stained
red by H and E stain) has invaded a
kidney glomerulus. Most cells are in
the yeast form, but some hyphae are
seen at the lower left. (Reproduced with
permission from Connor DH, Chandler
FW, Schwartz DQ, et al: Pathology
of Infectious Diseases. Stamford, CT:
Appleton & Lange, 1997.)
752 PART IV PAT H O G E N I C F U N G I

FIGURE 43–5. Fusarium invasion.


The branching septate hyphae are
stained black by this silver stain.
(Reproduced with permission from
Connor DH, Chandler FW, Schwartz DQ,
et al: Pathology of Infectious Diseases.
Stamford, CT: Appleton & Lange, 1997.)

The most commonly used medium for cultivating fungi is Sabouraud’s agar, which con-
Sabouraud’s agar optimal for tains only glucose and peptones as nutrients. Its pH is 5.6, which is optimal for growth of
fungi but poor for bacteria dermatophytes and satisfactory for growth of other fungi. Most bacteria fail to grow, or
grow poorly, on Sabouraud’s agar. A wide variety of other media are in use, many of which
use either Sabouraud’s or brain-heart infusion as their base.
Blood agar or other types of enriched bacteriologic media are used when pure cultures
would be expected, such as subculturing from blood cultures bottles in which yeast species
are observed. These media can be made more fungal-selective by the addition of antibacte-
rial antibiotics such as chloramphenicol and gentamicin. Cycloheximide, an antimicrobial
Selective media make use of that inhibits some saprophytic fungi, is sometimes added to Sabouraud’s agar to prevent
antimicrobials overgrowth of contaminating molds from the environment, particularly for skin cultures.
Media containing these selective agents cannot be relied on exclusively because they can
Cultures incubated at 30°C for interfere with growth of some pathogenic fungi or because the “contaminant” may be pro-
primary isolation ducing an opportunistic infection. For example, cycloheximide inhibits C neoformans, and
chloramphenicol may inhibit the yeast forms of some dimorphic fungi. In contrast to most
pathogenic bacteria, many fungi grow best at 25°C to 30°C, and temperatures in this range
are used for primary isolation. Paired cultures incubated at 30°C and 35°C may be used to
demonstrate dimorphism.
Once a fungus is isolated, identification procedures depend on whether the growing fun-
gus is a yeast or a mold. Yeasts are identified by biochemical tests analogous to those used
Yeast identified biochemically for bacteria, including some that are identical (eg, urease production). The observation of
specific fungal structures such as pseudohyphae is also diagnostically useful among the
yeasts.
Molds are most often identified by the morphology of their conidia and conidiophores.
Other features such as the size, texture, and color of the colonies help characterize molds,
Molds identified by morphology
but without demonstrating conidiation they are not sufficient for identification. The ease
and culture features
and speed with which various fungi produce conidia vary greatly. Minimal nutrition, mois-
ture, good aeration, and ambient temperature favor development of conidia.
Microscopic fungal morphology is usually demonstrated by methods that allow in situ
microscopic observation of the fragile asexual conidia and their shape and arrangement.
Morphology may also be examined in fragments of growth teased free of a mold and exam-
Lactophenol cotton blue stains
ined in preparations containing a dye called lactophenol cotton blue. The dye stains the
mycelia, conidia, and spores
hyphae, conidia, and spores. Conidium production may not occur for days or weeks after
the initial growth of the mold. It is similar to waiting for flowers to bloom, and it can be
frustrating when the result has immediate clinical application.
It is desirable, but not always possible, to demonstrate the yeast and mold phases with
Temperature variation demon- dimorphic fungi. In some cases, this result can be achieved with parallel cultures at 30°C
strates dimorphism and 35°C. The tissue form of C immitis is not readily produced in vitro. Demonstration of
dimorphism has become less important with the development of specific DNA probes for
DNA probes are more rapid the major systemic pathogens. These probes are rapid and can be applied directly to the
mycelial growth of the readily grown mold phases of these fungi.
CHAPTER 43 PAT H O G E N E S I S A N D D I A G N O S I S O F F U N G A L I N F E C T I O N S 753753

● Antigen and Antibody Detection


Serum antibodies directed against a variety of fungal antigens can be detected in patients
infected with those agents. These tests are rarely useful for diagnosing acute infections,
except for C immitis in which antibody levels often correlate with extent of infection. Immu-
Serologic tests are useful for
noassays to detect fungal antigens have been pursued for some time. The major targets
some fungi
are mannans, mannoproteins, glucan, chitin, or some other structure unique to the fungal
pathogen(s). Two of these tests are extremely sensitive and specific for systemic infection:
Antigen detection shows
(1) the C neoformans capsular polysaccharide antigen test (cryptococcal antigen) and (2)
promise
the H capsulatum surface antigen test (Histoplasma antigen). Serum antigen tests for Aspergillus
species (galactomannan) and other fungal pathogens (β-d-glucan) are less sensitive for
diagnosing infection but can be useful in some cases.
This page intentionally left blank
44
Antifungal Agents
and Resistance
Overview
Compared with antibacterial agents, relatively few antimicrobials are available for treatment
of fungal infections. Many substances with antifungal activity have proved to be unstable, Many antifungals are too toxic
to be toxic to humans, or to have undesirable pharmacologic characteristics, such as poor for use
diffusion into tissues. Newer antifungal agents target fungal-specific cellular features and
demonstrate lower toxicity than older drug classes.

M
any fungal infections are self-limiting and require no chemotherapy. Superficial
mycoses are often treated, but topical therapy can be used, thus limiting toxicity to
the host. Invasive or systemic fungal infections that are not controlled by the host’s
immune system require the prolonged use of antifungals. Given that most of the patients with Treatment is needed for invasive
these infections also have underlying immunosuppression, invasive mycosis can be among fungal infections in immunocom-
the most difficult of all infectious diseases to treat successfully. The characteristics of cur- promised persons
rently used antifungal agents are discussed next and summarized in Table 44–1. They are dis-
cussed in the text that follows in relation to their target of action, as illustrated in Figure 44–1.

ANTIFUNGAL AGENTS

CYTOPLASMIC MEMBRANE

● Polyenes
The polyenes nystatin and amphotericin B are lipophilic and bind to ergosterol, the domi-
nant sterol in the cytoplasmic membrane of fungal cells. After binding, they form annular ✺ Amphotericin B binds
channels, which penetrate the membrane and lead to leakage of essential small molecules ergosterol and forms membrane
from the cytoplasm and cell death. Their binding affinity for the ergosterol of fungal mem- channels
branes is not completely specific, cross-reacting with mammalian sterols such as choles-
terol. This is the basis for the considerable toxicity that limits their use. Almost all fungi are Active against most fungi
susceptible to amphotericin B, and the development of resistance is rare.
At physiologic pH, amphotericin B is insoluble in water and must be administered intra-
venously as a colloidal suspension. It is not absorbed from the gastrointestinal tract. The
major limitation to amphotericin B therapy is the toxicity created by its affinity for mamma-
lian as well as fungal membranes. Infusion is commonly followed by chills, fever, headache,
and dyspnea. The most serious toxic effect is renal dysfunction, observed in virtually every
patient receiving a prolonged therapeutic course. Experienced clinicians learn to titrate the

755
756 PART IV PAT H O G E N I C F U N G I

TABLE 44–1 Features of Antifungal Agents


AGENT ACTION RESISTANCE ROUTE CLINICAL USE
Polyenes
Nystatin Cell membrane pores Sterol modification Topical Most fungi
Amphotericin B Cell membrane pores Sterol modification Intravenous Aspergillus, Candida, Cryptococcus,
Histoplasma, Sporothrix, Coccidioides
Azoles
Fluconazole Ergosterol synthesis Efflux, demethylase Oral, intravenous Candida, Cryptococcus, Histoplasmac,
(demethylase) alteration, bypass, Coccidioidesc
overproductiona
Itraconazole Ergosterol synthesis Efflux, demethylase Oral, intravenous Aspergillus, Sporothrix, Candida,
(demethylase) alteration, bypass, Blastomyces, Histoplasma, Coccidioides
overproductiona
Voriconazole Ergosterol synthesis Oral, intravenous Candida, Aspergillus, some saprophytic
(demethylase) molds
Posaconazole Ergosterol synthesis Oral, intravenous Candida, Aspergillus (prophylaxis),
(demethylase) Zygomycetes
Isavuconazole Ergosterol synthesis Oral, intravenous Candida, Aspergillus (prophylaxis),
(demethylase) Zygomycetes
Clotrimazole Ergosterol synthesis Topical Candida, dermatophytes
(demethylase)
Ketoconazole Ergosterol synthesis Topical Candida, dermatophytes
(demethylase)
Miconazole Ergosterol synthesis Topical Candida, dermatophytes
(demethylase)
Allylamines
Terbinafine Ergosterol synthesis ?Efflux Oral, topical Dermatophytes
(squalene epoxidase)
Flucytosine (5-FC)
DNA synthesis, RNA Permease or modifying Oral Candida and Cryptococcusd
transcription enzymesb mutation
Echinocandins
Caspofungin Glucan synthesis Altered synthase Intravenous Candida, Aspergillus
(β-glucan synthase)
Micafungin Glucan synthesis Altered synthase Intravenous Candida, Aspergillus
(β-glucan synthase)
Anidulafungin Glucan synthesis Altered synthase Intravenous Candida, Aspergillus
(β-glucan synthase)
Nikkomycins Chitin synthesis Developmental
(chitin synthase)
Griseofulvin Microtubule disruption Unknown Oral Dermatophytes
Potassium iodide Unknown Unknown Oral Sporothrix schenckii

5FC, 5-Flucytosine.
a
Most work is with fluconazole and Candida; other azoles are to be assumed to be similar.
b
Cytosine deaminase and uracil phosphoribosyltransferase (the enzymes that modify 5-FC to active forms).
c
Itraconazole generally preferred.
d
Only in combination with amphotericin B owing to developing resistance

dosage for each patient to minimize the nephrotoxic effects. For obvious reasons, use of
Insoluble compound must be amphotericin B is limited to progressive, life-threatening fungal infections. In such cases,
infused in suspension despite its toxicity it retains a prime position in treatment often by administration of an
initial course of amphotericin followed by a less toxic agent. Preparations that complex
Therapy must be titrated against amphotericin B with lipids have been used as a means to limit toxicity. The even greater
toxicity toxicity of nystatin limits its use to topical preparations.
C H A P T E R 44 A N T I F U N G A L A G E N T S A N D R E S I S TA N C E 757757

Echinocandins

Glucan
Chitin
Nikkomycin

Azoles Ergosterol
Amphotericin B

FIGURE 44–1. Sites of action


Flucytosine
of antifungal agents. This figure
demonstrates the cellular targets of the
major antifungal agents: (1) cell wall
(echinocandins, nikkomycins), (2) cell
membrane (azoles, amphotericin B),
and (3) genome (flucytosine)

● Azoles
The azoles are a large family of synthetic organic compounds, which includes members
with antibacterial, antifungal, and antiparasitic properties. Their activity is based on inhibi-
tion of the enzyme (14 α-demethylase) responsible for conversion of lanosterol to ergos-
✺ Inhibit enzyme crucial
terol, the major component of the fungal cell membrane. This leads to ergosterol depletion
for synthesis of membrane
and lanosterol accumulation, forming defective membranes. All antifungal azoles have the
ergosterol
same mechanism of action. The differences among them are in avidity of enzyme binding,
pharmacology, and side effects. Azoles can also affect the precursors of some hormones and
may therefore cause endocrinological side effects, restricting their use in pregnancy.
The systemic azoles are generally grouped based on antifungal spectrum. Fluconazole
is primarily active against yeasts, including many Cryptococcus and Candida species. In
contrast, newer azole compounds have extended activity against molds such as Aspergillus
species, as well as the thermally dimorphic fungi (eg, Coccidioides, Blastomyces, Histo- Less toxic than amphotericin B
plasma). These mold-active azoles include itraconazole, voriconazole, posaconazole, and
isavuconazole. Most of these agents can be given orally or parenterally. Although generally Systemic azoles divided between
well-tolerated, the antifungal azoles can cause varying degrees of liver toxicity. Additionally, yeast-active and mold-active
all systemic azoles, except isavuconazole, can adversely affect cardiac myocyte repolariza- agents
tion and therefore prolong the QTc interval on an EKG, placing patients at increased risk
for cardiac arrhythmias. Topical agents such as ketoconazole, clotrimazole, and miconazole
are available in over-the-counter topical preparations for superficial mycoses.

● Allylamines
The allylamines are a group of synthetic compounds that act by inhibition of an enzyme
(squalene epoxidase) in the early stages of ergosterol synthesis. The allylamine group
✺ Inhibit ergosterol synthesis
includes an oral and topical agent, terbinafine used in the treatment of dermatophyte
(ringworm) infections.

NUCLEIC ACID SYNTHESIS

● Flucytosine
5-Flucytosine (5FC) is an analog of cytosine. It is a potent inhibitor of RNA and DNA syn- Enzymatically modified form
thesis. 5FC requires a permease to enter the fungal cell, where its action is not direct but makes defective RNA
through its enzymatic modification to other compounds (5-fluorouracil, 5-fluorodeoxyuridyic
acid, 5-fluoruridine). These metabolites then interfere with DNA synthesis and RNA ✺ Inhibits DNA synthesis
transcription.
758 PART IV PAT H O G E N I C F U N G I

Flucytosine is well absorbed after oral administration. It is active against most clinically
✺ Active against yeasts but not important yeasts, including Candida albicans and Cryptococcus neoformans, but it has little
molds activity against molds or dimorphic fungi. The frequent development of mutational resis-
tance during therapy limits its application to mild yeast infections or its use in combination
Resistance develops during with amphotericin B for cryptococcal meningitis. The primary toxic effect of flucytosine is
therapy if used alone a reversible bone marrow suppression that can lead to neutropenia and thrombocytopenia.
This effect is dose related and can be controlled by drug monitoring.

CELL WALL SYNTHESIS


The unique chemical nature of the fungal cell wall, with its interwoven layers of mannan,
glucan, and chitin (Figure 44–1), makes it an ideal target for chemotherapeutic attack. The
echinocandins, which block glucan synthesis, are now in clinical use and the nikkomycins,
which block chitin synthesis, are in development.

● Echinocandins
Echinocandins act by inhibition of a glucan synthetase (1,3-β-D-glucan synthetase) required
for synthesis of the principal cell wall glucan of fungi. Its action causes morphologic dis-
✺ Inhibits enzyme crucial for tortions and osmotic instability in yeast and molds, similar to the effect of β-lactams
glucan synthesis on bacteria. The first such agent to be licensed was caspofungin, which has good activity
against Candida and Aspergillus and a wide range of other fungi. Cryptococcus neoformans
Current indications for Candida, whose cell wall glucans have a slightly different structure is resistant. Since there are no
Aspergillus similar human structures, toxicity is minimal. The newest echinocandins, micafungin and
anidulafungin, have the same mode of action and a similar spectrum.

● Nikkomycins
Nikkomycins target fungal cell wall components similar to echinocandins. These com-
pounds inhibit chitin synthases, which polymerizes the N-acetylglucosamine subunits
that make up chitin. The result is inhibition of chitin synthesis. The agent in develop-
✺ Inhibits chitin synthesis
ment, nikkomycin Z, has activity against dimorphic fungi such as Coccidioides immitis
and Blastomyces dermatitidis but not against yeast or Aspergillus. However, chitin synthesis
remains an interesting strategy for novel antifungal development.

Other Antifungal Agents


Griseofulvin is a product of one of the Penicillium species of molds. Griseofulvin is actively
✺ Microtubule disruption taken up by susceptible fungi and acts on the microtubules and associated proteins that make
interferes with cell division up the mitotic spindle. It interferes with cell division and possibly other cell functions asso-
ciated with microtubules. Griseofulvin is absorbed from the gastrointestinal tract after oral
Active against dermatophytes administration and concentrates in the keratinized layers of the skin. It is active only against
the agents of superficial mycoses. Clinical effectiveness has been demonstrated for many causes
of dermatophyte infection, but the response is slow. Prolonged therapy may be required.
Potassium iodide is the oldest known oral chemotherapeutic agent for a fungal infec-
tion. It is effective only for cutaneous sporotrichosis. Its activity is somewhat paradoxical,
✺ Iodide inhibits Sporothrix because the mold form of the etiologic agent, Sporothrix schenckii, can grow on medium
containing 10% potassium iodide. The pathogenic yeast form of this dimorphic fungus
appears to be susceptible to molecular iodine.

RESISTANCE TO ANTIFUNGAL AGENTS

DEFINITION OF RESISTANCE
The concepts, definitions, and laboratory methods described in Chapter 23 for bacterial
resistance are generally applicable to fungi. Quantitative susceptibility is determined by
measuring the minimal inhibitory concentration (MIC) of a drug under conditions that
favor the growth of fungi. The wide range of growth rates and diversity of growth forms
C H A P T E R 44 A N T I F U N G A L A G E N T S A N D R E S I S TA N C E 759759

(yeast, hyphae, conidia) in the various fungi have added technical variables to testing, but Concepts of fungal resistance
standardized methods are now available. Comparison of MICs with drug pharmacology and susceptibility are similar to
allows classification of fungi as susceptible or resistant, but these results do not yet predict bacteria
clinical outcome with the same certainty they do with bacteria. Because of its specialized
nature, the availability of antifungal susceptibility testing is restricted to major centers and Fungal MICs are time-consuming
reference laboratories. and labor-intensive

MECHANISMS OF RESISTANCE
Many of the same resistance mechanisms observed in bacteria are also found in fungi.
Fungi tend to primarily inactivate drug activity using efflux pumps and by altering their
biosynthetic pathways. In contrast to bacteria, fungi do not make hydrolytic enzymes to
inactivate antibiotics to the same extent as bacteria. In part this may be due to the reduced
ability for horizontal gene transfer between species.

● Polyene Resistance
Because amphotericin B binds directly to the ergosterol in the fungal cell membrane, the Amphotericin B resistance is rare
only means to resist this action is to change the membrane sterol composition. Therefore, among fungi
only a few rare fungal species are intrinsically resistant to amphotericin B.

● Flucytosine Resistance
Flucytosine (5-FC) requires a permease for entry into the cell and then multiple enzymes
to modify it to the active metabolites that inhibit nucleic acid synthesis. Mutation in any
one of these enzymes renders the drug ineffective. This happens readily under the selective ✺ Multiple enzymes can mutate
pressure of 5-FC use. It is one of the few antimicrobials in which emergence of resistance to cause flucytosine resistance
is predictable during therapy of an acute infection. This is the reason that its use is mostly
limited to combination therapy with other antifungals.

● Azole Resistance
There are several mechanisms by which fungi can become resistant to the azoles. The most
well-characterized mechanism is through the induction of efflux pumps that transport the ✺ Azole pumped out by efflux
drug out of the cell. Some pumps act on all azoles, and others act on only one. Fungal spe- pumps
cies can also alter the subunits of the demethylase enzyme by mutation, effectively decreas-
ing the affinity of the azole for its enzyme target. Multiple mutations can have an additive Enzyme target altered
effect.
Additionally, some fungi are able to decrease the effect of the azoles by increasing the
production of the drug target. Some azole-resistant strains of Candida and Cryptococcus
species actually duplicate regions of their genome to increase the number of copies of the ✺ Demethylase enzyme
demethylase-encoding genes. This results in the requirement of higher concentrations of upregulated or bypassed
the azole drug to inhibit fungal growth. Some azole-resistant strains have also been shown
to accomplish ergosterol synthesis by an alternate pathway, thus bypassing the azole target
enzyme.

● Echinocandin Resistance
Although the echinocandins are relatively new, resistance has already been observed with
their use. One resistance mechanism is an altered target. Mutations in subunits of the glu-
✺ Mutant glucan synthetase
can synthetase have been correlated with increases in MIC more than a thousand-fold.

SELECTION OF ANTIFUNGALS
As with all chemotherapy, the selection of antifungal agents for treatment of superficial,
subcutaneous, and systemic mycoses involves balancing expected efficacy against toxic-
ity. The factors to be considered are the following: (1) the threat of morbidity or mortal-
ity posed by the specific infection, (2) the immune status of the patient, (3) the toxicity
of the antifungal, and (4) the probable activity of the antifungal agent against the fungus.
In the case of superficial mycoses, the risks of appropriate therapy are small, and various
760 PART IV PAT H O G E N I C F U N G I

topical agents may be safely used. At the other extreme, an immunocompromised patient
will most likely be treated aggressively with systemic agents for proven or even suspected
systemic fungal infection. Since susceptibility tests are usually not available, the decisions
regarding which agents to use are usually made and sustained on an empiric basis. Even
when guided by in vitro testing, treatment failures are common particularly in the immu-
nocompromised, emphasizing the prime importance of the immune system in preventing
and clearing systemic fungal infections. It is hoped that the addition of the novel antifungal
agents will have a favorable effect on these outcomes.
45
The Superficial and
Subcutaneous Fungi:
Dermatophytes,
Malassezia, Sporothrix,
and Pigmented Molds
Epidermophyton species Hortaea werneckii Cladophialophora
Microsporum species Sporothrix schenckii (Cladosporium) species
Trichophyton species Fonsecaea species
Malassezia furfur Phialophora species

T
he least invasive of the pathogenic fungi are the dermatophytes and other superfi-
cial fungi that are adapted to the keratinized outer layers of the skin. The subcuta-
neous fungi go a step farther by extending infection to the tissue beneath the skin
but rarely invading deeper structures (Table 45–1).

SUPERFICIAL FUNGI

Overview
Dermatophytoses are slowly progressive eruptions of the skin and its appendages.
Although often unsightly, they are not typically painful or life-threatening. The
manifestations vary depending on the site of infection and vigor of the host response,
but they often involve erythema, induration, itching, and scaling. The most familiar name
is “ringworm,” describing the annular shape of the advancing edge of this cutaneous
infection.

761
762 PART IV PAT H O G E N I C F U N G I

TABLE 45–1 Agents of Superficial and Subcutaneous Mycoses


FUNGAL GROWTH
FUNGUS IN LESION IN CULTURE (25°C) INFECTION SITE DISEASE
Dermatophytes
Microsporum canis Septate hyphae Mold Haira, skin Ringworm
Microsporum audouini Septate hyphae Mold Hair a
Ringworm
Microsporum gypseum Septate hyphae Mold Hair, skin Ringworm
Trichophyton tonsurans Septate hyphae Mold Hair, skin, nails Ringworm
Trichophyton rubrum Septate hyphae Mold Hair, skin, nails Ringworm
Trichophyton mentagrophytes Septate hyphae Mold Hair, skin Ringworm
Trichophyton violaceum Septate hyphae Mold Hair, skin, nails Ringworm
Epidermophyton floccosum Septate hyphae Mold Skin Ringworm
Other superficial fungi
Malassezia furfurb Yeast (mycelia)c Yeast Skin (pink to brown)d Pityriasis (tinea)
versicolor
Hortaea werneckiie Septate hyphae, Yeast (mold) Skin (brown to black)d Tinea nigra
ellipsoidal cells
Trichosporon cutaneum Septate hyphae Mold Hair (white)b White piedra
Piedraia hortae Septate hyphae Mold, ascospores Hair (black) b
Black piedra
Subcutaneous fungi
Sporothrix schenckii Cigar-shaped yeast (rare) Mold Subcutaneous, lymphatic spread Sporotrichosis
Fonsecaea pedrosoi Muriform body f
Mold Wart-like foot lesions Chromoblastomycosis
Phialophora verrucosa Muriform bodyf Mold Wart-like foot lesions Chromoblastomycosis
Cladophialophora Muriform body f
Mold Wart-like foot lesions Chromoblastomycosis
(Cladosporium) carrionii

a
Specimens fluoresce under ultraviolet light.
b
Previously known as Pityrosporum orbiculare.
c
Denotes less frequent findings.
d
Color of clinical lesions.
e
Previously known as Cladosporium werneckii.
f
Multicompartment yeast-like structure.

Dermatophytes
Dermatophytoses are superficial infections of the skin and its appendages. Common names
for these infections include ringworm (Figure 45–1), athlete’s foot, and jock itch. They are
caused by species of three genera collectively known as dermatophytes. These fungi are

FIGURE 45–1. Ringworm. The ring-


like lesions on this forearm are due
to advancing growth of Trichophyton
mentagrophytes. (Reproduced with
permission from Willey JM: Prescott,
Harley, & Klein’s Microbiology, 7th edition.
McGraw-Hill, 2008.)
CHAPTER 45 T H E S U P E R F I C I A L A N D S U B C U TA N E O U S F U N G I 763763

highly adapted to the nonliving, keratinized tissues of nails, hair, and the stratum corneum
of the skin. The source of infection may be humans, animals, or the soil.

MYCOLOGY
✺ Epidermophyton, Microsporum,
The three genera of medically important dermatophytes (literally, skin-plants) are and Trichophyton are major
Epidermophyton, Microsporum, and Trichophyton (Table 45–1). Most dermatophyte genera
infections are diagnosed and treated as a clinical syndrome since determining the
causative species will not usually affect therapeutic choices. Grow best at 25°C

DERMATOPHYTE DISEASE

Transmission can occur after close contact with an infected person or animal. However,
exposure to detached skin scales or hair containing the fungal elements (fomites) may also
✺ Transmission requires contact
result in new infections. Many places for common dermatophyte transmission have been
with intact or detached skin or
described, including locker rooms floors, barber shops, hotel carpets, and movie theater/
hair
airplane seats. Health-care workers do no need to take special precautions beyond hand-
washing after contact with an infected patient.

PATHOGENESIS
Dermatophytoses begin when the infecting fungus comes in contact with skin, especially if
there are minor breaks in the skin integrity. Detached hair and skin scales containing derma-
tophytes can remain infectious for months in the environment. Once the stratum corneum is
penetrated, the organism can proliferate in the keratinized layers of the skin, with a variety of ✺ Initial infection is through
proteinases helping to establish infection. The course of the infection depends on many factors: minor skin breaks
the anatomic location, the degree of skin moisture, the dynamics of skin growth and desquama-
tion, the speed and extent of the inflammatory response, and the infecting species. For example, Balance between fungal growth
if the organism grows very slowly in the stratum corneum and if skin turnover by desquama- and skin desquamation deter-
tion is rapid, the infection will probably be short-lived and cause minimal signs and symptoms. mines outcome
Inflammation tends to increase desquamation rates and helps to limit infection, whereas immu-
nosuppressive agents such as topical corticosteroids decrease shedding of the keratinized layers
and tend to prolong infection. Invasion of any deeper structures is extremely rare.
Most dermatophyte infections are self-limited, spontaneously resolving with time. How-
ever, those infections in which fungal growth rates and skin desquamation are balanced,
and in which the inflammatory response is poor, tend to become chronic. The lateral spread
Poor inflammatory response
of infection and its associated inflammation produce the characteristic sharp advancing
leads to chronic infection
margins that were once believed to be the burrows of worms. This characteristic is the ori-
gin of the common English name ringworm, as well as the Latin term tinea (worm), which
is often applied to the clinical forms of the disease (Figure 45–1).
Infection may spread from skin to other keratinized structures, such as hair and nails,
or may invade them primarily. The hair shaft is penetrated by hyphae (Figure 45–2), which ✺ Hair shaft is penetrated and
extend either exclusively within the shaft (endothrix) or both within and outside the shaft broken by hyphae
(ectothrix). The end result is damage to the hair shaft structure, which often breaks off. Loss
of hair at the root and plugging of the hair follicle with fungal elements may result. Inva- Infected nails are thickened and
sion of the nail plate causes a hyperkeratotic reaction, which dislodges or distorts the nail dislodged from nailbed
(onychomycosis).
Occasionally, dermatophyte infections become chronic and widespread. This progres-
sion has been related to host and organism factors. Approximately half of these patients
have underlying diseases affecting their immune responses or are receiving treatments
Widespread infection is associ-
that compromise T-lymphocyte function. These chronic infections are particularly associ-
ated with T-lymphocyte defects
ated with Trichophyton rubrum, to which both normal and immunocompromised persons
and T rubrum
appear to be hyporesponsive. Interestingly, the clinical manifestations of these infections
are largely due to delayed-type hypersensitivity responses to these agents rather than from
direct effects of the fungus on the host.
764 PART IV PAT H O G E N I C F U N G I

Hair shaft

Black piedra
FIGURE 45–2. Black piedra. Note
invasion by Piedraia hortae both within
(endothrix) and outside (ectothrix)
the hair shaft. Dermatophyte invasion
would be similar. (Reproduced with
permission from Willey JM: Prescott,
Harley, & Klein’s Microbiology, 7th edition.
McGraw-Hill, 2008.)

DERMATOPHYTOSES: CLINICAL ASPECTS

MANIFESTATIONS
Dermatophyte infections range from inapparent colonization to chronic progressive erup-
tions that last months or years, causing considerable discomfort and disfiguration. Derma-
tologists often give each infection its own “disease” name based on the Latin name for the
anatomic site at which the infection is found. For example, these names include tinea capitis
✺ Involved skin site defines the (scalp; Figure 45–3A), tinea pedis (feet, athlete’s foot), tinea manuum (hands), tinea cruris
type of “tinea” infection (groin), tinea barbae (beard, hair), and tinea unguium (nail beds). Skin infections otherwise
not included in this anatomic list are called tinea corporis (body). There are certain clini-
cal, etiologic, and epidemiologic differences among these syndromes, but they are basically
the same disease in different locations. The primary differences among etiologic agents that
infect different sites are shown in Table 45–1.

Tinea capitis. Infection of hair and scalp begins with an erythematous papule around the
hair shaft, which progresses to scaling of the scalp, and discoloration/fracture of the shaft.
Spread to adjacent hair follicles progresses in a ring-like fashion, leaving behind broken,

A B
FIGURE 45–3. Tinea capitis. A. Ringworm of the scalp with superficial lesions and loss of hair.
B. Close-up using an ultraviolet lamp (Wood’s light) reveals fluorescing hair fragments. The culture
grew Microsporum audouinii. (Reproduced with permission from Willey JM: Prescott, Harley, & Klein’s
Microbiology, 7th edition. McGraw-Hill, 2008.)
CHAPTER 45 T H E S U P E R F I C I A L A N D S U B C U TA N E O U S F U N G I 765765

discolored hairs, and sometimes black dots where the hair is absent but the infection has
Scalp infection (tinea capitis)
invaded the follicle. The degree of inflammatory response markedly affects the clinical
leads to itching and hair loss
appearance and, in some cases, can cause constitutional symptoms. In most cases, symp-
toms beyond itching are minimal.

Other sites. Skin lesions on other parts of the body begin in a similar manner and enlarge
to form sharply delineated erythematous patches with central clearing (nearly normal skin
appearance in the center). Multiple lesions can fuse to form unusual geometric patterns on
the skin. Lesions may appear in any location, but they are particularly common in moist, ✺ Skin infection favors moist
sweaty skin folds. Obesity and the wearing of tight apparel increase susceptibility to infec- areas and skin folds
tion in the groin and beneath the breasts. Another form of infection, which involves scaling
and splitting of the skin between the toes, is commonly known as athlete’s foot. Excessive
moisture and maceration of the skin provide the mode of entry.
Nail bed infections first cause discoloration of the subungual tissue, followed by hyper-
keratosis and discoloration of the nail plate. Progression of infection causes disfigurement
of the nail but few symptoms until the nail plate is so dislodged or distorted that it exposes ✺ Hyperkeratosis can dislodge
or compresses adjacent soft tissue. All dermatophyte infections provide a potential site of the nail plate
entry for skin bacteria, predisposing to more acute and painful lesions around the nail, or
to more extensive bacterial cellulitis.

Why is it especially important to treat tinea pedis in patients with diabetes


mellitus aggressively?

DIAGNOSIS
The goal of diagnostic procedures is to distinguish dermatophytoses from other causes of
skin inflammation. Infections caused by bacteria, other fungi, and noninfectious disorders
(eg, psoriasis and contact dermatitis) may have similar features. The most important step ✺ KOH mounts of skin scrapings
is microscopic examination of material taken from lesions to detect the fungus. Potassium and infected hairs demonstrate
hydroxide (KOH) or calcofluor white preparations of scales scraped from the advancing hyphae
edge of a dermatophyte lesion often demonstrate septate hyphae. Examination of infected
hairs reveals hyphae and arthroconidia penetrating the hair shaft. Broken hairs give the Some species fluoresce under uv
best yield. Some species of dermatophyte fluoresce when exposed to ultraviolet light, and light
selection of hairs for examination can be aided by the use of an ultraviolet (Wood’s) lamp
(Figure 45–3B).
The same material used for direct examination can be cultured for isolation of the offend-
ing dermatophyte (Figure 45–4). Mild infections with typical clinical findings and positive
KOH preparations are often not cultured because clinical management is not influenced
significantly by the identity of the etiologic species. Suspected dermatophyte infections Culture is used when KOH prepa-
with negative KOH preparations, especially those that fail to respond to empiric antifungal rations are negative
therapy, often require culture. The major reason for false-negative KOH results, however, is
failure to collect the scrapings or hairs properly.

TREATMENT AND PREVENTION


Many local skin infections resolve spontaneously without therapy. Those that do not resolve
may be treated with topical terbinafine or azoles (miconazole, ketoconazole). More exten-
sive skin infections, especially those involving the scalp, often require systemic therapy with

Think Apply 45-1. Many chronic changes occur in the skin of the feet of patients
with diabetes mellitus. Due to diabetes-associated microvascular changes, the skin can
become thin and dystrophic. In addition, diabetes-associated peripheral neuropathy
can prevent diabetic patients from noticing microtrauma to the skin that occurs during
daily activity. Together, these changes predispose many diabetic patients to develop
bacterial cellulitis of the lower extremities. This condition is more likely if the skin barrier
is compromised by tinea pedis, allowing bacteria ready access to deeper skin layers.
766 PART IV PAT H O G E N I C F U N G I

FIGURE 45–4. Large boat-shaped


macroconidia of Microsporum
gypseum. (Reproduced with
permission from Nester EW:
Microbiology: A Human Perspective,
6th edition. 2009.)
20 m

griseofulvin, itraconazole, or oral terbinafine, often combined with topical therapy. Nail
✺ Topical terbinafine or azoles infections are especially difficult to cure, likely due to the slow turnover of the infected nail
usually sufficient and poor penetration of antifungal agents. Therapy for nail infections must be continued
over weeks to months, and relapses may occur. Keratolytic agents (Whitfield’s ointment)
Systemic antifungal agents used may be useful for reducing the size of hyperkeratotic lesions. Dermatophyte infections can
in refractory cases usually be prevented simply by observing general hygiene measures. No specific preventive
measures such as vaccines exist.

● Other Superficial Mycoses


Pityriasis (tinea) versicolor is a very common superficial fungal infection of the skin. It is
characterized by discrete patches of either hypopigmentation or hyperpigmentation, espe-
cially on the skin of the torso and upper arms. These lesions are associated with some scal-
ing but minimal induration. Fungi of the genus Malassezia, especially Malassezia furfur,
M furfur requires lipids for
most commonly cause pityriasis versicolor. These fungal species are common components
growth
of the skin microbiome, but they are present more abundantly in the setting of clinical
infections. In scrapings of infected skin, they appear as clusters of budding yeast cells
mixed with hyphae. Malassezia species grow in the yeast form in culture media enriched
with lipids.
Tinea nigra, another superficial skin infection, is characterized by brown to black macu-
lar lesions, usually on the palms or soles. There is little associated inflammation or scaling,
and skin architecture is well preserved since the infection is confined to the stratum cor-
H werneckii causes superficial
neum. This feature distinguishes tinea nigra from other pigmented lesions such as mela-
pigmented lesions of hands and
nomas, which tend to change the lines and markings of the skin. This infection is caused
feet
by melanized, black-pigmented fungi (“dematiaceous” fungi) such as Hortaea werneckii,
commonly found in soil and other environmental sites. Scrapings of the lesion show brown
to black–pigmented septate hyphae.
Piedra is an infection of the hair characterized by black or white nodules attached to the
Black or white piedra are infec- hair shaft. White piedra (caused by Trichosporon cutaneum) infects the shaft in a hyphal
tions of hair shaft form that can fragment into component buds. Black piedra (caused by Piedraia hortae)
grows as branched hyphae in the hair shaft (Figure 44–2).

SUBCUTANEOUS FUNGI
Many fungal pathogens can produce subcutaneous lesions as part of their disease spectrum.
Those considered here are introduced traumatically through the skin, with infection typi-
cally limited to subcutaneous tissues, lymphatic vessels, and contiguous tissues. These fungi
rarely spread to distant organs. The diseases they cause include sporotrichosis, chromoblas-
tomycosis, and mycetoma. Only sporotrichosis has a single specific etiologic agent, Sporothrix
schenckii. Chromoblastomycosis and mycetoma are clinical syndromes with multiple fungal
etiologies.
CHAPTER 45 T H E S U P E R F I C I A L A N D S U B C U TA N E O U S F U N G I 767767

Sporothrix

Overview
Sporothrix schenckii is a dimorphic fungus widely present in soil and other organic matter.
Sporotrichosis begins with injection of the organism’s conidia into the subcutaneous
tissue. A thorn prick or splinter in the hand is a typical inciting event. Sporothrix schenckii
induces a slowly progressive infection that follows the lymphatic drainage from the original
site (lymphangitis). Superficial ulcers may occur, but the infection rarely involves deeper
structures.

SPOROTHRIX SCHENCKII

Sporothrix schenckii is a dimorphic fungus that grows as a cigar-shaped, 3- to 5-mm yeast ✺ Mold conidiophores convert
in tissues and in culture at 37°C. The mold, which grows in cultures incubated at 25°C, is to cigar-shaped yeast during
presumably the infectious form in nature. infection

SPOROTRICHOSIS

EPIDEMIOLOGY
Sporothrix schenckii is a ubiquitous saprophyte particularly found in hay, moss, soil (includ- ✺ Soil saprophyte is introduced
ing potting soil), and decaying vegetation, as well as the surfaces of various plants. Infection by trauma
is acquired by traumatic inoculation of the fungus through the skin. The skin of gardeners,
farmers, and rural laborers is frequently traumatized by thorns or other material that may ✺ Occupational disease of
be contaminated with conidia of S schenckii. gardeners and farmers

SPOROTRICHOSIS: CLINICAL ASPECTS

MANIFESTATIONS
Skin lesions due to S schenckii begin as painless papules developing a few weeks to a few
months after inoculation. Its location can usually be explained by occupational exposure;
the hand is most often involved. The initial papule enlarges slowly and eventually ulcerates,
leaving an open sore. Pustular or firm nodular lesions may appear around the primary site Skin papule eventually ulcerates
of infection or at other sites along the lymphatic drainage route (Figure 45–5). The spread
of infection along lymphatic channels is so characteristic for this infection that lymphan- ✺ Lymphatic involvement
gitic progression of any infection is often referred to as having a “sporotrichoid” appear- creates multiple lesions
ance. Ulcerated lesions can become chronic, and multiple ulcers develop if the disease is
untreated. Symptoms are directly related to the local areas of infection, with constitutional
signs and symptoms being unusual.

DIAGNOSIS
Direct microscopic examination for S schenckii is usually unrewarding because there are
too few organisms to detect readily with KOH preparations. Even specially stained biopsy
samples and serial sections are usually negative, although the presence of a histopathologic
structure, the asteroid body, is suggestive. This structure is composed of S schenckii yeast
cells surrounded by amorphous eosinophilic “rays.” Definitive diagnosis depends on culture
of infected pus or tissue. The organism grows within 2 to 5 days on all media commonly
used in medical mycology. Identification requires demonstration of the typical conidia and
of dimorphism.
768 PART IV PAT H O G E N I C F U N G I

A B
FIGURE 45–5. Sporotrichosis. A. This infection began on the finger and has started to spread up
the lymphatic channels of the arm, leaving satellite lesions behind. If untreated, these lesions will
evolve into ulcers. B. A more advanced case beginning with inoculation in the foot. (Reproduced
with permission from Connor DH, Chandler FW, Schwartz DQ, et al: Pathology of Infectious Diseases.
Stamford, CT: Appleton & Lange, 1997.)

TREATMENT AND PREVENTION


✺ Potassium iodide replaced by Historically, cutaneous sporotrichosis was treated with a saturated solution of potassium
itraconazole iodide (SSKI) administered orally. Itraconazole is now preferred for all forms of disease,
with oral terbinafine and SSKI as alternatives. Pulmonary and systemic infections may
Amphotericin B only for systemic require the additional use of amphotericin B. Eradication of the environmental reservoir of
disease S schenckii is not usually practical.

CHROMOBLASTOMYCOSIS
Chromoblastomycosis is a chronic form of skin infection caused by multiple species of
pigmented saprophytic fungi, also known as melanized or “dematiaceous” fungi. Disease
caused by Fonsecaea, Phialophora, and Cladophialophora (Cladosporium) species typically
occurs on the foot or leg, the sites of skin inoculation by the fungus. Primary lesions appear
as papules that develop into scaly, wart-like structures, usually under the feet. Fully devel-
oped lesions have been likened to the tips of a cauliflower. Extension occurs through pain-
Multiple species produce wart-
less, slowly progressive satellite lesions. The organisms are found in the soil of endemic
like pigmented lesions in tropics
areas, and most infections occur in individuals who work barefoot. Disseminated infec-
tions due to the dematiaceous molds are uncommon, observed almost exclusively in highly
immunocompromised patients. In fact, in these patients certain pigmented molds have a
predilection for infecting the central nervous system. Therefore, in highly immunocompro-
mised patients, the isolation of a pigmented mold from a clinical specimen cannot neces-
sarily be ignored as a contaminant.
The outstanding mycologic feature in chromoblastomycosis is the presence in histologi-
cal sections of brown-pigmented, thick-walled, multiseptate, 5 to 12 mm globose struc-
Brown pigmented bodies are tures called muriform bodies. Branching septate hyphae may also be demonstrated in KOH
seen in chromoblastomycosis preparations of tissue scrapings. In culture, these fungi grow as darkly pigmented molds.
Surgery and antifungal therapy have been used in chromoblastomycosis, but cure rates of
advanced disease are disappointing. Flucytosine or itraconazole are the systemic antifungal
agents most frequently used for this infection.

MYCETOMA
Mycetoma is the clinical term for a chronic, disfiguring infection associated with prior
trauma to the foot. Varied microorganisms cause this infection, with more than a dozen
fungi described as potential etiologies. Bacterial actinomycetes such as Nocardia
CHAPTER 45 T H E S U P E R F I C I A L A N D S U B C U TA N E O U S F U N G I 769769

(see Chapter 28) may produce a similar disease. The typical clinical appearance of
mycetoma is that of massive induration with draining sinuses. Some of the fungi that
cause mycetoma are geographically widespread; most cases, however, occur in the tropics, ✺ Mycetoma—massive
probably because of the chronically damp, macerated skin of the feet among those who inflammatory lesions of feet with
go barefoot in tropical environments. An illustrative case of mycetoma occurred in a draining sinuses
college rower in Seattle; he was the only member of his shell who insisted on rowing
barefoot.
Multiple species are involved
Once infection is established, treatment of mycetoma is difficult, often requiring com-
bined surgical and antimicrobial therapy depending on the causative microbe. Mycetomas
Trauma to bare feet injects the
caused by environmental molds may demonstrate hyphae in biopsied tissue. However,
fungi
these fungal elements may be difficult to demonstrate because of a tendency to form micro-
colony granules. The definitive diagnosis of the etiological agent often requires culture of
infected tissue.

KEY CONCLUSIONS
• Dermatophytes infect the superficial, keratinized layers of the skin.
• Dermatophyte infections are generally diagnosed as a clinical syndrome, named
for the infected body site (eg, tinea capitis, tinea pedis).
• Nail infections caused by dermatophytes are especially difficult to treat for cure.
• Pityriasis (tinea) versicolor manifests as patches of hypo- or hyperpigmented
lesions of the skin of the torso and proximal extremities.
• Sporotrichosis begins by inoculation of the skin with soil or plant material, and it
often spreads along regional lymphatics as nodules and ulcers.

CASE STUDY
HEAD BUMP
A 4-year-old boy was taken by his mother to the family doctor for evaluation of
a 2-month history of a slowly growing “bump” on the back of his head. The boy
had no other siblings or any pets at home. He attended a day-care center each
weekday while his mother worked. Examination revealed a happy, alert child in no
distress. A raised, scaling lesion 3.5 cm in diameter with a few pinpoint pustules
was present on the posterior scalp. A KOH preparation of material from the lesion
was negative. A fungal culture of material from the lesion was later positive for
a fungus with numerous microconidia and macroconidia typical of Microsporum
species.

QUESTIONS
1. What is the most likely source of this child’s infection?
A. Parents
B. Child at day care center
C. Animal
D. Insect
E. Food

2. What is the human niche where this organism proliferates best?


A. Fibronectin
B. Macrophages
C. M cells
D. Keratin
770 PART IV PAT H O G E N I C F U N G I

3. What additional examination might have revealed this infection while the child
was in the doctor’s office?
A. X-ray
B. Serologic test
C. Ultraviolet light
D. Biopsy
E. DNA probe

ANSWERS
1. (B), 2. (D), 3. (C)
46
The Opportunistic Fungi:
Candida, Aspergillus,
the Zygomycetes, and
Pneumocystis
Candida albicans Candida parapsilosis Mucor species
Candida glabrata Aspergillus fumigatus Pneumocystis jirovecii
Candida krusei Absidia species
Candida tropicalis Rhizopus species

T
he “opportunistic fungi” are usually found as members of the resident human
microbiota or as saprophytes in the environment. With the breakdown of host
defenses, they can cause infections ranging from skin/mucous membrane involve-
ment to life-threatening, systemic disease. The most common opportunistic infections are
caused by two species: the yeast Candida albicans, a common inhabitant of the gastroin-
testinal and genital microbiota; and the mold Aspergillus fumigatus which is widespread
in the environment. Pneumocystis, a frequent cause of pneumonia in AIDS patients, is an
unusual fungus that used to be considered a parasite on morphologic grounds. However,
it too is a frequent colonizer of the human respiratory tract. The diseases caused by these
opportunistic fungi are summarized in Table 46–1.

CANDIDA: GENERAL CHARACTERISTICS


Candida species grow as 4 to 6 μm, budding, round or oval yeast-like cells (Figure 46–1)
under most conditions and at most temperatures. Under certain growth conditions, includ-
ing those encountered during infection, certain pathogenic Candida species can also form
hyphae. Of the over 150 Candida species, fewer than 10 cause human infections. Particular
attention is given to the differentiation of C albicans from other species, because it is by far
the most common cause of disease. For serious infections, identifying Candida isolates to
the species level is important for prognostic and treatment decisions (Figure 46–1).

771
772 PART IV PAT H O G E N I C F U N G I

TABLE 46–1 Agents of Opportunistic Mycoses


GROWTH
ORGANISM TISSUE CULTURE AT 25°C CULTURE AT 37°C SOURCE INFECTION
Candida Yeast (hyphae) a
Yeast (hyphae) a
Yeast Endogenous Skin, mucous membranes,
urinary, disseminated
Aspergillus Hyphae (septate) Mold Mold Environment Lung, disseminated
Zygomycetesb Hyphae (nonseptate) Mold Mold Environment Rhinocerebral, lung,
disseminated
Pneumocystis Elliptical spores Nonec Nonec Unknown Pneumonia

a
Less common feature; pseudohyphae are produced as well.
b
Most common genera are Absidia, Mucor, and Rhizopus.
c
Has not been grown in culture.

Overview
Candida species are budding yeasts that readily grow in culture. Candidiasis occurs in
localized and disseminated forms. Localized disease is seen as erythema and white plaques
in moist skinfolds (diaper rash) or on mucosal surfaces (oral thrush). It may also cause the
itching and thick white discharge of vulvovaginitis. Candida bloodstream and urinary
tract infections are especially common among hospitalized patients with intravenous and
urinary catheters. Deep tissue and disseminated infections are limited almost exclusively to
the immunocompromised.

MYCOLOGY
Candida species grow in multiple morphologic forms, most often as a budding yeast.
Candida albicans, the most common cause of human invasive fungal infections, is also able
✺ C albicans grows with varied to form hyphae triggered by changes in conditions such as temperature, pH, and available
morphologies: yeast, hyphae, nutrients. When observed in their initial stages of germination from the yeast cell, these
and pseudohyphae nascent hyphae resemble sprouts and are called “germ tubes” (Figure 46–2A). Other elon-
gated forms with restrictions at regular intervals are called pseudohyphae because they lack
the parallel walls and septation of true hyphae. There is evidence that these three forms have

FIGURE 46–1. Candida albicans.


This scanning electron micrograph
demonstrates dimorphism with both
yeast-like blastoconidia and hyphae.
(Reproduced with permission from
Willey JM: Prescott, Harley, & Klein’s
Microbiology, 7th edition. McGraw-Hill,
2008.)
C H A P T E R 46 THE OPPORTUNISTIC FUNGI 773773

FIGURE 46–2. Candida albicans.


A. When incubated at 37°C, C albicans
rapidly forms elongated hyphae
called germ tubes. B. On specialized
media, C albicans forms thick-walled
chlamydoconidia, which differentiate it
from other Candida species. (Reprinted
with permission from Dr. E. S. Beneke
and the Upjohn Company: Scope
A B Publications, Human Mycoses.)

distinct stimuli and genetic regulation, making C albicans a polymorphic fungus. Unless
otherwise specified, the term hyphae is used here to encompass both the true hyphal and
pseudohyphal forms. Being able to recognize these different fungal morphologies can help
clinicians and laboratory personnel to rapidly distinguish C albicans from other, similar
yeast species in clinical specimens.

CANDIDIASIS

EPIDEMIOLOGY
Candida albicans is present in the microbiota of 30% to 50% of healthy persons, especially
common in the oropharyngeal, gastrointestinal, and female genital tracts. Infections are
endogenous, with most symptoms arising from infections with one’s own resident species.
However, transmission and new acquisition of Candida colonization can occur by direct
✺ C albicans infections are often
mucosal contact with others (eg, through sexual intercourse). Similarly, in nosocomial
from endogenous flora
C albicans infections, the strains involved are usually derived from the patient’s own flora
rather than from cross-infection. Invasive procedures and indwelling devices may provide
the portal of entry, and the number of Candida present on the skin and mucosal surfaces
may be enhanced by the use of antibacterial agents.

PATHOGENESIS
Because C albicans is regularly present on mucosal surfaces, disease implies a change in the
organism, the host, or both. The ability of this microorganism to change between the yeast
and hyphal forms is strongly associated with its pathogenic potential, and these different
morphological forms are likely required for different phases of candidiasis. In histologic
preparations, hyphae are seen during Candida invasion, either superficially into the mucosa Shift from yeast to hyphae is
or within deep tissues. However, dissemination in the bloodstream is likely enhanced dur- associated with invasion
ing the yeast growth phase. Therefore, it is the plasticity between the two forms, rather than
one morphology or the other, that results in the ability of C albicans to so effectively colonize Switch is triggered by
and infect the host. The yeast-hyphal switch can be controlled in vitro by the manipulation environmental conditions
of a wide variety of environmental conditions (serum, pH, temperature, amino acids). Vari-
ous sensors and signaling pathways for morphogenesis have been described including one
in which C albicans induces its own morphological change by directly altering the local pH.
Candida albicans hyphae have the capacity to form strong attachments to human epi-
thelial cells. One mediator of this binding is a surface hyphal wall protein (Hwp1), which
is found only on the surface of germ tubes and hyphae. Other surface mannoproteins that
have similarities to vertebrate integrins may also mediate binding to components of the
774 PART IV PAT H O G E N I C F U N G I

Fibronectin binding Proteinase activity

Fibronectin Hyphae

Epithelial cell ECM binding

Basement Collagen IV
membrane Laminin
Entactin

ECM
Interstitial
collagen

FIGURE 46–3. Pathogenesis of Candida albicans infections. Proposed mechanisms of


C albicans attachment and invasion are shown. Surface glucomannan receptor(s) on the yeast may
bind to fibronectin covering the epithelial cell or to elements of the extracellular matrix (ECM) when
the epithelial surface is lost or when the Candida cells have invaded beyond it. Invasion is associated
with formation of hyphae and production of proteinases, which may digest tissue elements.

extracellular matrix (ECM), such as fibronectin, collagen, and laminin. Hyphae also secrete
proteinases and phospholipases that are able to digest epithelial cells and probably facili-
tate invasion (Figures 46–3 and 46–4A and B). One family of hyphal enzymes, the secreted
aspartic proteinases (Saps), is able to digest keratin and collagen, which would facilitate
deep tissue invasion. The pattern of Sap production may be tissue-specific with individual
isolates producing distinct sets of Saps depending whether they are invading gastrointes-
tinal or vaginal epithelium. One of the most important pathogenic features of C albicans
is its ability to form biofilms. These complex structures include yeast and hyphal forms of
the fungus along with host-derived proteins. Once formed, the biofilm strongly adheres
to components of the ECM as well as to plastics. Neither host immune cells nor antifun-
gal agents are able to penetrate Candida biofilms well, making this structure an important
source of microbial persistence during infection. In a very practical sense, fungal biofilms
that develop on prosthetic surfaces (eg, intravenous catheters, prosthetic joints, prosthetic
heart valves) are almost impossible to sterilize without device removal.
Many factors predispose to both local and invasive Candida infections. Antibacterial
therapy reduces microbial competition on mucosal surfaces and increases the relative abun-
dance of C albicans within the microbiota. Alterations in innate immunity (eg, leukopenia

A B
FIGURE 46–4. Invasiveness of Candida albicans. Two features of invasiveness are seen in these
scanning electron micrographs taken from experiments with murine corneocytes. A. Both yeast-like
blastoconidia and mycelial elements are present. The mycelial elements spread over the surface and
invade the cell cuticle. B. A C albicans strain that produces a protease is seen producing cavity-like
depressions in the cell surface. This action could play a role in invasion of the cell. (Reprinted with
permission of Thomas L. Ray and Candia D. Payne. Infect Immunol. 1988;56:1945-1947, Figures 4,6B.
Copyright American Society for Microbiology.)
C H A P T E R 46 THE OPPORTUNISTIC FUNGI 775775

or corticosteroid therapy) or adaptive immunity (eg, AIDS) are important contributing fac-
tors to systemic and mucosal candidiasis. Additionally, anatomic disruptions of the skin Antimicrobials and immunosup-
and mucosa may enhance the invasion process by exposing Candida binding sites in the pression increase risk
ECM, and by allowing direct access to deeper tissues. Biofilm formation on the plastics
used in medical devices also contributes to fungal persistence in this host. Diabetes mellitus Mechanical disruptions may
predisposes to C albicans infection, possibly due to greater production of surface manno- provide access to ECM
proteins in the presence of high glucose concentrations.

IMMUNITY
Both humoral immunity and cell-mediated immunity (CMI) are involved in defense
against Candida infections. Neutrophils are the primary first-line defense, and defects in
neutrophil number or function are among the most common immune correlates of seri-
ous C albicans infections. Yeast forms of C albicans are readily phagocytosed and killed Opsonized yeast forms are killed
by these innate immune cells, especially when opsonized by antibody and complement. by PMNs
Hyphal forms may be too large to be ingested by polymorphonuclear neutrophils (PMNs),
but these immune cells can still kill the fungi by attaching to the hyphae and discharging
metabolites generated by the oxidative burst.
Many immunodeficiency syndromes involving T-lymphocyte dysfunction result in
severe mucocutaneous candidiasis, emphasizing the importance of this arm of the immune
system in defense against Candida infections. For example, patients with AIDS develop Compromised CMI is associated
frequent episodes of oral and esophageal candidiasis, suggesting that protection against with progressive infection
even superficial infections involves CD4-mediated immune responses. As with other fungi,
cytokine activation of macrophages and other immune effector cells enhances their ability Balance between TH1 and TH2
to kill C albicans. Therefore, a favorable outcome of infection appears to require the proper cytokines is necessary
balance between TH1- and TH2-mediated cytokine responses, and this balance is lost in the
absence of intact CD4 cell function.

CANDIDIASIS: CLINICAL ASPECTS

MANIFESTATIONS
Superficial invasion of the mucous membranes by C albicans produces a white, cheesy
plaque that is loosely adherent to the mucosal surface. Oral lesions, called thrush, occur
on the tongue, palate, and other mucosal surfaces as ragged white patches (Figure 46–5).

FIGURE 46–5. Thrush. The white


plaques on this AIDS patient’s tongue
are caused by Candida albicans.
(Reproduced with permission from
Willey JM: Prescott, Harley, & Klein’s
Microbiology, 7th edition. McGraw-Hill,
2008.)
776 PART IV PAT H O G E N I C F U N G I

A B
FIGURE 46–6. Candida albicans skin infection. A. This rash is preceded by chronically damp skin
in the diaper area. B. This Gram stain demonstrates yeast cells and pseudohyphae. (Reproduced with
permission from Nester EW: Microbiology: A Human Perspective, 6th edition. 2009.)

Scraping the fungal plaque with a tongue blade will reveal varying degrees of underlying
✺ White mucosal plaque is called mucosal invasion and inflammation, helping to differentiate this infectious process from
thrush other causes of superficial oral films. A similar infection in the vagina, vaginal candidiasis,
produces a thick, curd-like discharge and itching of the vulva. Although many women
✺ Vaginitis may be recurrent have at least one episode of vaginal candidiasis in a lifetime, a small proportion suffers
chronic, recurrent infections. No general or specific immune defect has yet been linked to
this syndrome.
Superficial C albicans infections also occur in skinfolds and other areas in which wet,
macerated skin surfaces are opposed. For example, one type of diaper rash is caused
✺ Macerated skin is a common by C albicans (Figure 46–6A). Other infections of the skinfolds and appendages occur in
site of Candida skin infections association with recurrent immersion in water (eg, dishwashers). The initial lesions are ery-
thematous papules or confluent areas of erythema, tenderness, and skin fissures. Infection
usually remains confined to the chronically irritated area with adjacent “satellite” lesions.
Rarely, chronic and relapsing Candida infections occur in patients with a specific defect
in TH1 immune defenses. This condition, known as chronic mucocutaneous candidiasis
(CMC), manifests with recurrent severe skin and mucosal lesions. With time, patients with
CMC experience considerable skin disfigurement. Although lesions may become extensive,
they usually do not result in fungal dissemination.
In contrast, most people live their entire lives in constant contact with C albicans but
without developing symptomatic infections. This observation largely reflects the ability of
Chronic mucocutaneous candi- normal hosts to effectively control the growth of this fungus. However, Candida coloniza-
diasis is associated with specific tion without associated symptomatic inflammation may also represent a clinical example of
T-cell defects immunologic tolerance—to be able to be exposed to a microorganism without developing
an excessive immune response. Therefore, it is possible that a subset of patients with fre-
quent episodes of symptomatic candidiasis (eg, recurrent vulvovaginal candidiasis) actually
has difficulty suppressing an overexuberant immune reaction to resident fungi, as opposed
to trouble controlling microbial growth.
Inflammatory patches similar to those in thrush may also develop in the esophagus and
✺ Esophagitis and intestinal upper GI tract. These lesions occur most frequently in immunocompromised patients and
candidiasis are similar to thrush are characterized by painful swallowing or substernal chest pain. Extensive ulcerations,
deformity, and occasionally perforation of the esophagus may ensue.
In addition to infection of mucosal surfaces, Candida infection often involves the urinary
tract. Ascending infections may produce cystitis, pyelonephritis, renal abscesses, or expand-
ing fungus ball lesions in the renal pelvis. Patients with urinary catheters, kidney trans-
plants, or other types of chronic urinary devices are at particular risk for these infections.
Urinary tract infections are Disseminated infections are particularly serious forms of candidiasis. The fungus often
ascending or hematogenous gains access to the bloodstream through skin lesions (eg, burns, intravascular catheters),
disruption of the GI tract (eg, intestinal perforations, abdominal surgery), or prosthetic
Endophthalmitis appears as devices colonized with Candida biofilms. Once in the bloodstream, Candida species can
white cotton-like retinal lesions infect many organs, including the kidneys, brain, and heart valves; however, symptoms are
C H A P T E R 46 THE OPPORTUNISTIC FUNGI 777777

generally not sufficiently characteristic to suggest C albicans over the bacterial pathogens.
Importantly, disseminated candidiasis frequently involves the eye. Candida endophthalmitis has
the characteristic funduscopic appearance of a white cotton ball expanding on the retina or
floating free in the vitreous humor. Endophthalmitis and infections of other eye structures
can lead to blindness, and ocular complications must be considered in every case of
disseminated candidiasis.

What specific interventions might limit the incidence of mucosal candidiasis


(oral thrush) and vaginal candidiasis in immunologically normal hosts?
What specific interventions might limit the incidence of candidemia in hos-
pitalized patients?

DIAGNOSIS
Exudate or epithelial scrapings examined by KOH preparations (Figure 46–6B) demon- KOH and Gram smears of super-
strate abundant budding yeast cells; if associated hyphae are present, the infection is almost ficial lesions show yeast and
certainly caused by C albicans. C albicans is readily isolated in culture from clinical speci- hyphae
mens including blood. Cultures from specimens, such as sputum, run the risk of contami-
nation from yeasts present in the normal flora. ✺ Candida species are readily
Deep organ involvement is difficult to prove without a direct aspirate or biopsy. However, grown in routine cultures of
Candida species often grow in routine blood cultures, and every episode of candidemia must clinical samples
be carefully evaluated for evidence of dissemination of involvement of prosthetic devices.

TREATMENT
Candida albicans is usually susceptible to amphotericin B, nystatin, flucytosine, the echi-
nocandins, and the azoles. Superficial infections are generally treated with topical nystatin
or azole preparations. Measures to decrease moisture and chronic trauma are important
✺ Topical nystatin or azoles for
adjuncts in treating Candida skin infections. All C albicans infections may also require
superficial lesions
addressing predisposing conditions. For example, removal of an infected catheter, control
of diabetes, or an increase in peripheral leukocyte counts can be important aspects of the
✺ Amphotericin B, fluconazole,
complete treatment of infection. Systemic therapy with amphotericin B, echinocandins, or
and echinocandins for invasive
azoles is required for disseminated or deep tissue infections. The choice of treatment is
disease
often guided by speciation and antifungal susceptibility testing. Fluconazole has been effec-
tive treatment for chronic mucocutaneous candidiasis and recurrent mucosal infections,
although antifungal resistance can develop with the prolonged use of this agent.

Other Candida Species


Candida species other than C albicans can produce very similar infections to those described
earlier, especially disseminated and urinary tract infections. However, these nonalbicans
Candida species are isolated almost exclusively from patients with nosocomial infections.
Antibiotic use, wounds, and prosthetic devices also predispose hospitalized patients to
✺ Candida glabrata and Candida
infections with diverse Candida species. Some species, such as C glabrata and C krusei, dis-
krusei are often resistant to
play increased levels of resistance to the azole antifungals. Therefore, selection for coloniza-
azoles
tion by these species may occur in patients previously treated with azoles. Other species,
such as C tropicalis and C parapsilosis, are also isolated mostly from hospitalized patients.
Importantly, nosocomial transmission of Candida species may occur with poor adherence
to proper handwashing and other infection control practices.

Think Apply 46-1. Many cases of oral thrush could be prevented by limiting
unnecessary antibacterial use, avoiding the associated disruption of the mucosal
microbiota that predisposes to yeast overgrowth. Candidemia can be limited by
minimizing unnecessary intravenous and urinary catheter use in hospitalized patients.
In health care settings, adherence to good hand-hygiene limits spread from person to
person.
778 PART IV PAT H O G E N I C F U N G I

ASPERGILLUS

Overview
Aspergillus species are molds found readily in the environment. Invasive aspergillosis
occurs in highly immunocompromised persons, often rapidly resulting in death. The
typical patient with this infection is neutropenic due to treatments for leukemia or perhaps
undergoing hematopoietic stem-cell transplantation. Fever and a dry cough may be the
only clinical signs of early aspergillosis, even before pulmonary infiltrates are demonstrated
radiologically. Finding definitive evidence for Aspergillus infections is challenging, and
many patients are treated empirically for this condition in the appropriate clinical setting.

MYCOLOGY

Aspergillus species are rapidly growing molds that are frequently isolated from the envi-
ronment. Since we are all exposed continuously to this group of fungi, it is not surprising
that they are common causes of infections in patients with severe immune compromising
conditions.
In culture and during infection, Aspergillus species grow as branching septate hyphae.
With prolonged culture in the laboratory, fruiting bodies develop as sites of spore (conidia)
Species are distinguished based formation. Individual species are defined on the basis of differences in the structure of the
on arrangement of conidia on conidiophore and the arrangement of the conidia. (Figure 46–7A to C). The most common
the conidiophore infections in humans are caused by A fumigatus, but others species, such as A flavus, A niger,
and A terreus may infrequently cause human disease. For all of these species, fluffy colonies
appear in 1 to 2 days; by 5 days, they may cover an entire plate with hyphal growth.

ASPERGILLOSIS

EPIDEMIOLOGY
Aspergillus species are widely distributed in nature and throughout the world. They seem
to adapt to a wide range of environmental conditions, and the heat-resistant conidia pro-
vide a good mechanism for dispersal. Like bacterial spores, the fungal conidia survive well
in the environment, with inhalation as the mode of human infection. Outbreaks of pul-
Conidia may be spread through monary aspergillosis in hospitals have been traced to fungal spores transmitted through
air ducts air ducts, emphasizing the potential for environmental acquisition of infection. Similarly,
building construction and remodeling have also been associated with increased frequency
of Aspergillus infections. Hospital wards with highly immunocompromised patients often
use specialized air filtration devices to limit patient exposure to these environmental fungi.

PATHOGENESIS
Once inhaled, Aspergillus conidia are small enough to readily reach the terminal airways
and alveoli. However, the resident innate immune cells are able to suppress the germina-
tion of these cells into growing hyphae. Therefore, lung infections due to Aspergillus species
are exceeding rare in people with normal immune systems. Host factors favoring Aspergillus
growth include anatomic abnormalities of the lungs (eg, bronchiectasis, severe emphysema)
and immunosuppression (neutropenia, lung transplantation).

IMMUNITY
Aspergillus infection is extremely rare in healthy persons. Macrophages, particularly pulmo-
✺ Alveolar macrophages kill nary alveolar macrophages, are the first line of defense against inhaled Aspergillus conidia,
conidia, and PMNs attack hyphae phagocytosing and killing them by nonoxidative mechanisms. For the conidia that survive
C H A P T E R 46 THE OPPORTUNISTIC FUNGI 779779

A B

C
FIGURE 46–7. Aspergillus. A. This asexual conidium-forming structure is characteristic of
Aspergillus species. The conidia are borne at the end of the finger-like extensions at the end of the
conidiophore. These structures are rarely produced in vivo. B. This tissue aspirate mixed with KOH
shows branching, septate hyphae. C. Histologic sections also show branching, septate hyphae, but
because the conidia shown in A are not seen, the findings are not diagnostic of Aspergillus. (A and
C, Reproduced with permission from Connor DH, Chandler FW, Schwartz DQ, et al: Pathology of
Infectious Diseases. Stamford, CT: Appleton & Lange, 1997.)

and germinate, PMNs become the primary defense. They are able to attach to the growing
hyphae, generate an oxidative burst, and secrete reactive oxygen intermediates.

ASPERGILLOSIS: CLINICAL ASPECTS

MANIFESTATIONS
Aspergillus infections typically present in one of four major ways, with the clinical find-
ings completely dependent on the immune status of the host. These clinical presentations
include: (1) Aspergillus pneumonia, (2) disseminated aspergillosis, (3) allergic respiratory
disease, and (4) aspergilloma (fungus ball).
Aspergillus pneumonia. As the main site of fungal cell entry into the body, the lung is the
primary organ involved in most cases of aspergillosis. Although innate immune responses
in the lung are usually able to clear Aspergillus spores as they are inhaled, anatomic and
immune defects can allow this fungus to grow and establish infection. Patients with severe
forms of emphysema and bronchiectasis have regions of anatomic abnormalities within
their lungs that offer protected sites for fungal germination and growth. Resident macro-
phages and neutrophils in the lung are less effective at controlling fungal growth in these
780 PART IV PAT H O G E N I C F U N G I

regions of scarring within the airways and lung parenchyma. Therefore, patients with
emphysema or cystic fibrosis can develop microinvasive lung infections due to Aspergillus
species. In the presence of an intact immune system, the infection is unable to deeply
penetrate into the surrounding lung tissue or to disseminate. However, this smoldering
infection can result in symptoms of chronic bronchopneumonia, with flares of cough and
worsening respiratory function.
In contrast, patients with severe defects of immunity, such as solid-organ transplanta-
tion or neutropenia, can develop a progressive and immediately life-threatening pneu-
monia due to Aspergillus species. This rapidly progressive infection does not require prior
lung anatomic abnormalities, and it emphasizes the importance of the innate immune
✺ Pulmonary aspergillosis in system in normally preventing fungal colonization and growth. During invasive pulmo-
immunocompromised patient is nary aspergillosis, the fungal hyphae penetrate intact lung tissue, leading to local necro-
highly invasive sis (Figure 46–7C). Symptoms include fever, cough, hemoptysis, and respiratory failure.
Rapidly progressive lung infiltrates are often observed radiographically. Untreated, this
infection quickly leads to the death of the patient. Even in the presence of antifungal
therapy, cure of this infection often requires restoration of immune function (eg, recovery
from neutropenia).
Disseminated aspergillosis. Once a primary Aspergillus infection is established in an
immunocompromised patient, the fungus can disseminate through the bloodstream to
affect any organ system in the body. The most dreaded site of spread is the central nervous
system. The symptoms of disseminated infection will depend on the organs affected. How-
ever, changes in mental status, or other specific organ dysfunction, should always prompt
a thorough investigation in immunosuppressed patients who are at risk for this devastat-
ing complication. When feasible, surgical debridement of infected tissue may favor patient
survival. However, disseminated aspergillosis carries a high mortality rate. Therefore, much
effort has gone into developing rapid diagnostic and preventive strategies in neutropenic
patients, since delays in the treatment of invasive aspergillosis are associated with worse
survival.
Allergic respiratory disease. As ubiquitous components of the air that we breathe, fun-
gal spores are typically captured by the mucus present in our respiratory tract. In patients
with allergic tendencies, the accumulation of excessive mucus in the airways may provide
a growth substrate for inhaled fungi. After germinating, these fungal elements are not able
to invade the underlying respiratory tissue, nor are they able to disseminate from this ini-
tial growth site. However, as potent allergens, the fungi may induce a cycle of progres-
sive inflammation creating enhanced mucinous substrate for fungal growth. The growth
of environmental Aspergillus species in the sinuses and larger airways can cause chronic
allergic symptoms in susceptible patients.
Of note, children with severe forms of asthma are especially prone to develop a condition
known as allergic bronchopulmonary aspergillosis (ABPA), characterized by eosinophilia,
symptomatic asthma flares, cough, and respiratory distress. Anti-Aspergillus antibodies
Allergic disease marked by can also be detected in the bloodstream. During flares of ABPA, fleeting infiltrates may be
eosinophilia and specific anti- seen on chest radiographs. These infiltrates do not reflect invasive fungal disease, but rather
Aspergillus IgG the extensive inflammation due to the allergic reaction to fungal antigens. This condition
is treated with antiallergy therapy (eg, inhaled/systemic corticosteroids, antihistamines)
rather than antifungal therapy. Allergic sinus disease due to fungi is thought to be an espe-
cially common cause of chronic sinusitis.
Aspergilloma (fungus ball). Patients with prior lung infections can develop pulmonary
scarring and cavities. Historically, this was especially common in patients with old, healed
tuberculosis. Fungal spores that found their way into these cavities were often able to grow
into macroscopic fungal colonies. These “balls” of fungal hyphae appear radiographically
as round regions of consolidation inside of old lung cavities. Unable to penetrate into the
surrounding tissues, these fungus balls nonetheless are often able to induce mechanical
Fungus ball in lung cavities
trauma to the wall of the lung cavity. Patients with aspergillomas therefore often experi-
ence recurrent episodes of hemoptysis. If large blood vessels are present near these cavi-
ties, life-threatening bleeding can occur. Antifungal therapy may help some patients with
this condition. However, cavity removal or catheter-guided embolization of involved lung
vessels may also be required for recurrent hemorrhages due to aspergillomas.
C H A P T E R 46 THE OPPORTUNISTIC FUNGI 781781

DIAGNOSIS
Aspergillus can be isolated and identified in cultures of infected tissue. Its rapidly spread-
ing mold growth, and all too frequent contamination of cultures, cause it to be regarded
Direct aspirate or biopsy
by microbiologists as a kind of weed. The diagnostic problem is distinguishing Aspergillus
required to distinguish coloniza-
contamination and colonization from invasive disease. The diagnosis cannot be made for
tion from invasion
certain without the use of lung aspiration, biopsy, or bronchoalveolar lavage. Fungal growth
from infected material demonstrates the presence of large, branching, septate hyphae
Serodiagnosis is useful only for
(Figure 46–7B and C). Detection of circulating Aspergillus antigens (galactomannan, glu-
allergic disease
can) by immunoassay or nucleic acid amplification techniques shows promise, but cross-
reactions with other fungi have been a problem.

Why would rapid and noninvasive diagnostic tests be important advances


for invasive aspergillosis?

TREATMENT AND PREVENTION


The mold-active azoles, especially voriconazole, are the preferred treatments for invasive
aspergillosis. Caspofungin and amphotericin B are alternatives. No regimen is considered ✺ Voriconazole, amphotericin
highly effective because the mortality rate of invasive disease is high. Surgical removal of B, caspofungin, and surgery for
localized lesion is sometimes helpful, even in the brain. Construction of rooms with filtered invasive aspergillosis
air has been effective in reducing exposure to environmental conidia.

ZYGOMYCETES AND ZYGOMYCOSIS/MUCORMYCOSIS


Absidia, Rhizopus, and Mucor are
Zygomycosis (mucormycosis) is the term applied to infection with any of a group of zygo-
soil saprophytes
mycetes, the most common of which are Absidia, Rhizopus, and Mucor. These fungi are
ubiquitous saprophytes in soil and are commonly found on bread and many other food-
✺ Immunocompromised hosts
stuffs. They occasionally cause disease in persons with diabetes mellitus and in immunosup-
and those with diabetes are
pressed patients receiving corticosteroid therapy. Diabetic ketoacidosis has a particularly
infected
strong association with zygomycosis.
Pulmonary or rhinocerebral disease is acquired by inhalation of conidia. The pulmonary
form has clinical findings similar to those of other fungal pneumonias. The rhinocerebral
form, however, produces a dramatic clinical syndrome in which agents of zygomycosis show Pulmonary disease is similar to
striking invasive capacity. They penetrate the mucosa of the nose, paranasal sinuses, or pal- that from other fungi
ate, often resulting in ulcerative lesions. Once beyond the mucosa, they progress through tis-
sue, nerves, blood vessels, fascial planes, and often the vital structures at the base of the brain. Sinus infections erode straight to
The clinical syndrome begins with sinus symptoms and headache and may rapidly progress the brain
through orbital cellulitis and hemorrhage to cranial nerve palsy, vascular thrombosis, coma,
and death.
The pathologic findings in mucormycosis are distinctive: the zygomycetes involved all show
ribbon-like, nonseptate (aseptate) hyphae in tissue which are so large that their branch points
can be difficult to visualize (Figure 46–8). Conidia are not seen in tissue. As with Aspergillus,
tissue biopsies are necessary to demonstrate the invasive hyphae, unless they can be seen on
✺ Large ribbons of aseptate
scrapings from palatal or nasal ulcers. For reasons that are obscure, cultures are sometimes
hyphae are seen in tissues
negative, even those from tissue containing characteristic hyphae. Therapy involves control
of underlying disease (eg, recovery from neutropenia, treatment of hyperglycemia) and high-
dose antifungal therapy (lipid-associated amphotericin B, selected azoles). Surgical debride-
ment is the most important intervention favoring survival and cure of infection.

Think Apply 46-2. Due to the devastated immune system of most patients with
invasive aspergillosis, this infection can progress rapidly, leading to patient death.
Culture-based diagnostic tests are rarely positive in this infection, perhaps due to
poor access to adequate specimens. Patients with neutropenia (low neutrophils) are
also frequently thrombocytopenic (low platelets), making clinicians hesitant to biopsy
infected tissue due to concerns for hemorrhage. Delays in diagnosis and treatment of
invasive aspergillosis are associated with poor survival.
782 PART IV PAT H O G E N I C F U N G I

FIGURE 46–8. Zygomycosis. This


zygomycete has invaded a blood
vessel. Note the ribbon-like hyphae
without septation. (Reproduced with
permission from Connor DH, Chandler
FW, Schwartz DQ, et al: Pathology
of Infectious Diseases. Stamford, CT:
Appleton & Lange, 1997.)

PNEUMOCYSTIS

Overview
Pneumocystis has not been grown in culture and is placed with the fungi due its chemical
components and molecular analysis. Pneumocystis pneumonia is an opportunistic infection
with nonspecific initial symptoms, such as fever or malaise. Respiratory symptoms come
later with nonproductive cough and shortness of breath. Radiographs reveal symmetric
alveolar pulmonary infiltrates, which spread outward from the hilar regions. Progressive
hypoxia can lead to death in a 3- to 4-week period. The observation of an increased
incidence of Pneumocystis pneumonia in the early 1980s was one of the first clinical clues
to the identification of HIV/AIDS.

Pneumocystis jirovecii is a ubiquitous colonizer of the human airway. It does not cause infec-
tions in most people, but it can cause a lethal pneumonia in immunocompromised persons,
particularly those with AIDS. The more familiar name to many clinicians, Pneumocystis
✺ rRNA and mitochondrial gene carinii, is now use for a Pneumocystis species found in rats. Pneumocystis jirovecii has not
sequences homologous with been grown in culture and was long considered a parasite rather than a fungus based on the
fungi morphology of forms seen in infected tissue. The designation of Pneumocystis as a fungus is
most strongly supported by genome sequencing. This molecular phylogenic analysis places
Pneumocystis among the ascomycetes.

MYCOLOGY

✺ Life cycle is deduced from Because it has not been possible to cultivate Pneumocystis, our knowledge about its basic
static images biology is limited. Observations rest on the study of organisms purified from infected lungs
and genomic analysis of Pneumocystis DNA. The Pneumocystis “life cycle” is deduced from
✺ Cysts in tissue appear like static images seen in infected tissues. The most commonly observed form in tissue is a
“crushed ping pong balls” delicate 5 to 8 μm cystic structure (Figure 46–9). No filamentous form has been observed.

PNEUMOCYSTOSIS

EPIDEMIOLOGY
Pulmonary infection with Pneumocystis occurs worldwide in humans and in a broad spec-
Worldwide distribution in
trum of animal life. Pneumocystis is not typically observed in the respiratory tract of asymp-
humans and animals
tomatic persons, but it was detected in one study by nucleic acid amplification techniques
C H A P T E R 46 THE OPPORTUNISTIC FUNGI 783783

FIGURE 46–9. Pneumocystis


pneumonia. A silver stain of this
material from the lung reveals folded
cysts some of which contain comma-
shaped spores. (Reproduced with
permission from Connor DH, Chandler
FW, Schwartz DQ, et al: Pathology
of Infectious Diseases. Stamford, CT:
Appleton & Lange, 1997.)

in over half the fatal victims of automobile accidents. Among HIV-infected individuals, the Airborne transmission is
strains involved in second and third episodes are frequently antigenically different, suggest- probable
ing frequent reacquisition of new strains.
Before the AIDS pandemic, Pneumocystis pneumonia (PCP) occurred sporadically
among infants with congenital immunodeficiencies and in older children and adults as a
complication of immunosuppressive therapy. Now AIDS has become the most common
predisposing condition, and PCP is often the presenting manifestation of AIDS. In fact, ✺ PCP is a complication of
before the development of effective chemoprophylactic regimens (see Treatment and Pre- immunodeficient states
vention), PCP occurred in approximately 50% of all AIDS patients at the time of initial
diagnosis. Depending on the effectiveness of their HIV treatment regimen, AIDS patients ✺ AIDS patients are at high risk
may develop one or more bouts of PCP, often in conjunction with another opportunistic
infection.

PATHOGENESIS
Pneumocystis is an organism of low virulence, which seldom produces disease in a host
with normal T-lymphocyte function. In experimental animals, progressive infection can be
Low CD4 counts increase the risk
initiated with starvation or corticosteroid administration, and in AIDS patients the risk of
in AIDS
developing pneumocystosis increases dramatically once the CD4+ T-lymphocyte count has
fallen below 200 cells/mm3. Concurrent viral, bacterial, fungal, and protozoan infections
are found frequently in humans with PCP, suggesting that Pneumocystis may require the
presence of another microbial agent for its multiplication.
Histologically, PCP is characterized by alveoli filled with desquamated alveolar cells,
monocytes, organisms, and fluid, producing a distinctive foamy, honeycombed appearance Alveoli filled with foamy exudate
(Figure 46–10); hyaline membranes may be present, and round cell infiltrates may be visible
in the septa.

IMMUNITY
The nature of the immunodeficiencies in patients with pneumocystosis points to the pri-
macy of TH1 immune responses in resolution of infection with Pneumocystis. Alveolar mac- Activated macrophages and
rophages are the first line of defense, with activated macrophages and CD4+ lymphocytes cytokines mediate CMI
playing essential roles in the resolution of the infection. Activated macrophages release sev-
eral cytotoxic factors, including O2-derived radicals, reactive nitrogen intermediates, and
cytokines (tumor necrosis factor-α, IL-2).
784 PART IV PAT H O G E N I C F U N G I

FIGURE 46–10. Lung biopsy specimen


from Pneumocystis pneumonia,
showing “foamy” contents of alveoli.
(Reproduced with permission from
Connor DH, Chandler FW, Schwartz DQ,
et al: Pathology of Infectious Diseases.
Stamford, CT: Appleton & Lange, 1997.)

PNEUMOCYSTOSIS: CLINICAL ASPECTS

MANIFESTATIONS
In the immunocompromised host, the disease presents as a progressive, diffuse pneumoni-
tis. Illness may begin after discontinuation or a decrease in the dose of corticosteroids or,
in the case of acute lymphocytic leukemia, during a period of remission. These observa-
tions suggest that the immune response to the organism results in many of the symptoms
✺ Diffuse pneumonitis with accompanying the infection. In infants and patients with AIDS, symptom onset is typically
insidious onset insidious, and the clinical course is often 3 to 4 weeks in duration. Fever is mild or absent.
In older persons and patients who have previously been on high doses of corticosteroids,
Nonproductive cough, dyspnea, the onset can be more abrupt. In both populations, the cardinal manifestations are progres-
and cyanosis develop later sive dyspnea and tachypnea; cyanosis and hypoxia eventually supervene. A nonproductive
cough is present in 50% of all patients. Clinical signs of pneumonia are usually absent,
Alveolar infiltrates spread out despite the presence of infiltrates on X-ray. These infiltrates are alveolar in character and
from the central regions of the spread out symmetrically from the hilar regions of the lungs, eventually affecting most of
lungs the lung. Occasionally, unilateral infiltrates, coin lesions, lobar infiltrates, cavitary lesions,
or spontaneous pneumothoraces are observed. Pleural effusions are uncommon. Clinical
and radiographic abnormalities are generally accompanied by a decrease in arterial oxygen
saturation, diffusion capacity of the lung, and vital capacity. Death occurs by progressive
asphyxia.
Lesions outside the lung were rarely seen before the AIDS epidemic, but they now appear
with some regularity in this patient population. The sites most often involved are lymph
Extrapulmonary lesions are seen nodes, bone marrow, spleen, liver, eyes, thyroid, adrenal glands, gastrointestinal tract, and
in AIDS kidneys. The extrapulmonary clinical manifestations range from incidental autopsy find-
ings to progressive multisystem disease.

DIAGNOSIS
Diagnostic yield from sputum is Definite diagnosis of pneumocystosis depends on finding organisms of typical morphology
low in appropriate specimens. Because the pathologic process is alveolar rather than bronchial,
the organisms are not readily seen in expectorated specimens such as sputum. The diagnos-
BAL is the best of the invasive tic yield is much better from specimens obtained by more invasive procedures. Of these,
procedures bronchoalveolar lavage (BAL) gives the best results with the least morbidity.
C H A P T E R 46 THE OPPORTUNISTIC FUNGI 785785

Pneumocystis can be demonstrated by a variety of staining procedures. The standard


stain is methenamine silver (Figure 46–9), but direct fluorescent antibody (DFA) method, ✺ Silver and other stains readily
if available, is slightly more sensitive. Methods developed for detection of Pneumocystis demonstrate P carinii
DNA in BAL and other specimens by polymerase chain reaction may soon be practical
for clinical laboratories. The detection of fungal antigens in the blood, such as β-D glucan, DFA is sensitive
supports the diagnosis.

To prevent missed diagnoses, should tests for PCP be routinely added to


standard clinical microbiological panels used to evaluate respiratory sam-
ples from patients with pneumonia?

TREATMENT AND PREVENTION


The fixed combination of trimethoprim and sulfamethoxazole (TMP-SMX) is the treatment
of choice for all forms of pneumocystosis. It is administered orally or intravenously for 14
to 21 days. Patients with AIDS receive the longer course because they start with a higher ✺ TMP-SMX is treatment of
organism burden, respond more slowly, and suffer relapse more often. Adjunctive cortico- choice
steroids in hypoxic patients with PCP results in improved survival, likely due to prevention
of an excessive immune reaction to dying microbes.
Low-dose administration of TMP-SMX has been shown to significantly decrease the
incidence of PCP in high-risk patients and prevents relapse in patients with AIDS. This
chemoprophylaxis is indicated for patients who have CD4+ lymphocyte counts lower than Chemoprophylaxis prevents PCP
200/mm3, unexplained fever, or a previous episode of PCP. Chemoprophylaxis for PCP is in high-risk groups
also used in solid-organ transplant patients immediately after transplantation and during
treatment for allograft rejection.

KEY CONCLUSIONS
Candida
• Candida albicans is a commensal yeast-like fungus in the normal, human mucosal
microbiota.
• Growing in both yeast and hyphal forms, C albicans can form nearly impenetrable
biofilms on prosthetic material, as well as causing mucosal and disseminated
infections.
• Certain Candida species are resistant to specific antifungals. Therefore, the
particular Candida isolate causing systemic infections should be identified to the
species level.
• Bloodstream infections due to Candida species are very common in hospitalized
patients. When present, consider potential sources (eg, IV catheters, intestinal
lesions) as well as potential distant sites of spread (eg, retina).
Aspergillus
• Invasive aspergillosis occurs most commonly in patients with defective neutrophils.
• Immediate antifungal therapy and surgical debridement are important
interventions when IA is considered or recognized in immunocompromised
patients.
• Environmental controls and prophylactic antifungal therapy may decrease the
incidence of IA for selected patient populations (eg, hematopoietic stem-cell
transplant patients).
Mucormycosis/zygomycosis
• Environmental molds such as the zygomycetes can cause destructive sinus
infections in patients with poorly controlled diabetes mellitus.
• Pulmonary infections occur in highly immunocompromised patients and have a
high mortality.

Think Apply 46-3. Clinicians should maintain a high index of suspicion for PCP in
patients with known or suspected defects in cell-mediated immunity (AIDS, transplant
patients). This infection is exceedingly uncommon in immunocompetent patients,
limiting the effectiveness of general screening for PCP in most cases of pneumonia.
786 PART IV PAT H O G E N I C F U N G I

Pneumocystis
• Pneumocystis pneumonia (PCP) occurs in patients with profound defects in CD4+
lymphocyte function.
• In addition to antimicrobial therapy, adjunctive corticosteroids may decrease
mortality in severe cases of PCP by limiting immune-mediated damage during
antimicrobial treatment.

CASE STUDY
A BUDDING BLOOD CULTURE
This 71-year-old woman was admitted with a recurrence of poorly differentiated
squamous cell carcinoma of the cervix. She underwent extensive gynecologic
surgery (excision of the organs of the anterior pelvis) and was maintained
postoperatively on broad-spectrum intravenous antibiotics. The woman had a
central venous catheter placed on the day of the surgery.
Beginning 3 days postoperatively, the patient had temperatures of 38.0°C
to 38.5°C, which persisted without a clear source. On post-op day 8, she had a
temperature of 39.2°C. Cultures of blood and of the tip of the central line both
grew an agent with large ovoid cells, some of which had constricted buds at their
ends. When incubated in serum, these cells sprouted long tubes with parallel
sides.

QUESTIONS
1. Which organism is most likely to be identified in this patient’s blood culture?
A. Candida albicans
B. Candida glabrata
C. Aspergillus
D. Mucor
E. Pneumocystis
2. What feature of the organism might have facilitated its infection in these
circumstances?
A. Mannoprotein
B. Glucan
C. Germ tube formation
D. Biofilm formation
E. Sporocytes
3. Which is the probable origin of the infecting agent?
A. Animals
B. Hospital air
C. Medical devices
D. Patient’s flora
E. Healthcare workers

ANSWERS
1. (A), 2. (D), 3. (D)
47
The Systemic Fungal
Pathogens: Cryptococcus,
Histoplasma, Blastomyces,
Coccidioides,
Paracoccidioides
Cryptococcus neoformans/ Blastomyces dermatitidis Paracoccidioides
Cryptococcus gattii Coccidioides immitis/ brasiliensis
Histoplasma capsulatum Coccidioides posadasii

T
he fungi discussed in this chapter cause a variety of infections, each ranging in severity
from subclinical to progressive, debilitating disease. Some of these species are dimor-
phic, growing in the infectious mold form in the environment but switching to a round,
yeast-like form in infected tissues. They differ from the opportunistic fungi in their ability to
cause disease in previously healthy persons. However, the most serious infections still occur in
patients with compromised immune systems. With the exception of Cryptococcus neoformans,
each of these fungi is restricted to geographic niches corresponding to the environmental
habitats of the mold form of the species. None of these infections is transmitted from human
to human. The major features of the systemic pathogens are summarized in Table 47-1.

CRYPTOCOCCUS

Overview
The Cryptococci are yeasts distinguished by a surrounding capsule. The primary disease
caused by cryptococci is a chronic meningitis. The clinical onset is slow, even insidious,
with low-grade fever and headache progressing to altered mental state and seizures. In
the cerebrospinal fluid (CSF) and in tissues, the inflammatory response is often remarkably
muted. Most patients who develop this infection have some obvious form of immune
compromise, although some show no demonstrable immune defect.

787
788 PART IV PAT H O G E N I C F U N G I

TABLE 47–1 Features of Systemic Fungal Pathogens


GROWTH
CULTURE AT CULTURE AT PRIMARY DISSEMINATED
ORGANISM 25°C 37°C TISSUE SOURCE DISEASE DISEASE
Cryptococcus Encapsulated Encapsulated Encapsulated Environment, Pneumonia Chronic meningitis
neoformans, C gattii yeast yeast yeast worldwide
Histoplasma Mold, tuberculate Small yeast Small intracel- Environment, Pneumonia, hilar RES enlargement
capsulatum macroconidiaa lular yeastb US Midwestd adenopathy
Blastomyces Molda Yeast Environment, Pneumonia Skin and bone
dermatitidis US Midwestc lesions
Coccidioides Mold, (Spherules)e Spherules Environment, Valley fever Pneumonia, menin-
immitis, C posadasii arthroconidia Sonoran desertc,f gitis, skin, and bone
Paracoccidioides Mold Yeast, multiple Environment, Pneumonia Mucocutaneous,
brasiliensis blastoconidia Latin America RES

RES, reticuloendothelial system (lymph nodes, liver, spleen, and bone marrow).
a
Micoconidia are formed but are not distinctive.
b
Typically multiple yeast within macrophages.
c
Ecologic “islands” are found throughout the Americas.
d
Ecologic islands are found worldwide.
e
It is difficult to grow the spherule phase in culture.
f
In the United States and includes parts of Arizona, California, Nevada, and western Texas.

CRYPTOCOCCUS NEOFORMANS AND CRYPTOCOCCUS GATTII


Cryptococcus species were first isolated from environmental sources more than a century
ago, and they are now recognized as important human pathogens, especially in the setting
of HIV infection. The most important clinical manifestation of cryptococcal disease is a
life-threatening meningitis in immunocompromised patients.
Found throughout the world, Cryptococcus species grow primarily as a budding yeast 4
to 6 μm in diameter. The most characteristic feature of these cells is a large polysaccharide
capsule (Figure 47–1), often extending the overall diameter of these cells to 25 μm or more.
The Cryptococcus genus contains two pathogenic species complexes, C neoformans and the
more recently recognized C gattii.
The cryptococcal capsule is a unique feature among pathogenic fungi, composed of a
complex polysaccharide polymer. The major components of the capsule are glucuronoxylo-
mannan and glucuronoxylomannogalactan, together referred to as GXM. Capsule produc-
tion is repressed under environmental growth conditions, and it is stimulated by the human
physiologic conditions found in tissues and in culture on some laboratory media. Capsular
material is also secreted into the surrounding environment, serving to suppress the activity
of nearby immune cells. GXM is so potently immunosuppressive that it has been used to
treat autoimmune conditions such as rheumatoid arthritis in experimental trials.

FIGURE 47–1. Cryptococcus


neoformans. This India ink preparation
was made by mixing cerebrospinal
fluid containing cryptococci with India
ink. The yeast cells can be seen within
the clear space caused by the large
polysaccharide capsule excluding
the ink particles. Note that the one
on the right is budding. (Reproduced
with permission from Nester EW:
Microbiology: A Human Perspective,
6th edition. 2009.) 20 m
CHAPTER 47 T H E S Y S T E M I C F U N G A L PAT H O G E N S 789789

CRYPTOCOCCOSIS

EPIDEMIOLOGY
Cryptococcus neoformans can be isolated from environmental samples throughout the
world, particularly in soil contaminated with bird droppings and decaying vegetable mat-
ter. The infectious form is felt to be either desiccated yeast cells or basidiospores stirred up
from these sites and subsequently inhaled. The less common species C gattii was once felt to
be restricted to tropical and subtropical area, but it has recently been isolated from patients
and the environment near the US Pacific Northwest (British Columbia, Washington, and
Oregon).
Cases of symptomatic cryptococcal infections in immunologically normal people are
very rare, although it is well known that most people are exposed to this fungus early in
childhood. This suggests that cryptococcal species are well controlled by the immune sys-
tem after initial infection. Cryptococcosis in immunocompromised patients occurs pri-
marily in those with defects in T-lymphocyte function, particularly in patients with AIDS,
✺ Associated with soil and bird
in whom it is the most common systemic fungal infection. Recent data estimated that more
droppings
than 500,000 deaths occur each year in AIDS patients due to this infection. In countries
with well-developed antiretroviral therapy programs, the incidence of cryptococcal disease
Inhaled yeasts or basidiospores
has markedly declined in recent years. However, this infection remains an important clini-
initiate the infection
cal issue in other immunocompromised populations. Life-threatening disease can occur in
patients with no known immune defects, although many clinicians believe that poorly char-
acterized immune disorders may explain the majority of these infections. Person-to-person
transmission has not been documented, with most cases likely resulting from reactivation
of dormant foci of remote infections, similar to tuberculosis.

PATHOGENESIS
After being inhaled, cryptococci reach the alveoli, where production of the polysaccharide
capsule is the prime determinant of virulence. The capsule is antiphagocytic and has vari-
ous other immunomodulating effects, such as downregulation of cytokines, interference
with antigen presentation, inhibition of leukocyte migration, misdirecting of specific anti-
body responses, and delaying the development of TH1 immune responses. These immune- ✺ “Crypto” is immunologically
suppressing effects may act at both a local and systemic level because cryptococci produce “hidden” behind its capsule
sufficient capsule that the GXM is readily detected in the blood and other body fluids. If
engulfed by macrophages, C neoformans is able to survive and multiply by altering its meta-
bolic pathways and by inducing melanin production, which interferes with oxidative killing
mechanisms.
The affinity of C neoformans for the central nervous system (CNS) is striking. Proposed ✺ Antiphagocytic capsule is
explanations include crossing the blood–brain barrier inside macrophages (Trojan horse prime factor
model) and the ability of laccase to convert the abundant catecholamines in the CNS to
melanin. Similar to other neuropathogens, components on the microbial surface may also ✺ Circulating GXM interferes
help to target C neoformans to the CNS by a specific interaction with proteins on the endo- with immune function
thelial cells of the brain microvasculature.
Melanin provides oxidative
protection in macrophage

CRYPTOCOCCOSIS: CLINICAL ASPECTS

MANIFESTATIONS
Meningitis is the most commonly recognized form of cryptococcal disease. Unlike bacterial
✺ Meningitis is insidious and
infections of the CNS, cryptococcal meningitis usually has a slow, insidious onset with rela-
chronic
tively nonspecific findings until late in its course. Common presenting symptoms include
intermittent headache, irritability, dizziness, and difficulty with complex cerebral functions,
✺ Course is more rapid with AIDS
appearing over weeks or months. Behavioral changes have sometimes been mistaken for
790 PART IV PAT H O G E N I C F U N G I

psychoses. Fever is usually, but not invariably, present. Seizures, cranial nerve defects, and
papilledema may appear later in the clinical course, as may dementia and decreased levels
of consciousness. A more rapid course may be seen in AIDS patients, and 5% to 15% of
these patients may have symptomatic infection at some point. Although the onset of illness
may be subacute, cryptococcal infection of the CNS is usually fatal if not recognized and
treated.
Like many other pathogenic fungi that enter the host through the lung, most initial
pulmonary infections are minimally symptomatic. Infections can be truly clinically inap-
parent, or they may manifest as a self-limited respiratory illness. However, cryptococcal
pneumonia can be progressive and severe in immunocompromised patients. In either case,
no clinical findings are sufficiently specific to suggest the etiology.
Dissemination of infection occurs almost exclusively in immunocompromised patients,
sometimes targeting the skin and bones. Classically, cryptococcal skin lesions are papular
or nodular, often with a central umbilication and remarkable for their lack of inflammation.
The diagnosis is sometimes made when lesions are biopsied as suspected neoplasms.
There is evidence of differences in the disease spectrum of the two Cryptococcus spe-
Cryptococcal pneumonia often cies. Cryptococcus gattii is more likely to produce pulmonary infection and less likely to
asymptomatic invade the CNS. Cryptococcus gattii infection has also been described more frequently
in patients with no definable immunological defect. In the CNS, C gattii may cause more
CNS involvement varies with localized lesions (cryptococcomas) as opposed to the diffuse meningoencephalitis typical
species of C neoformans.

DIAGNOSIS
Typical CSF findings in cryptococcal meningitis are increased intracranial pressure, pleo-
cytosis (usually more than 100 white blood cells/mm3) with predominance of lymphocytes,
Cells and glucose depression in and depression of glucose levels. In some cases, one or all of these findings may be absent,
CSF may be minimal yet cryptococci are still isolated on culture. Encapsulated yeast cells (diagnostic of
C neoformans infection) are demonstrable in CSF in approximately 50% of cases by mixing
India ink preparation is positive centrifuged CSF sediment with India ink and examining the mixture under the microscope
in 50% of cases (Figure 47–1). Experience is necessary to avoid confusion of lymphocytes with crypto-
cocci. C neoformans stains poorly or not at all with routine histologic stains; thus, it is easily
missed unless special fungal stains are used (Figure 47–2).
For the isolation of C neoformans by culture, the volume of CSF sampled is important.
The number of organisms present may be small enough to require a substantial volume of
Few cryptococci may be present fluid (more than 30 mL) to yield a positive culture. The detection of cryptococcal capsular
in CSF antigen in the CSF is the most sensitive and specific way to make the diagnosis of crypto-
coccal meningitis. This test can also be performed on serum where it is especially useful in
diagnosing infection in immunocompromised patients due to higher levels of circulating
✺ GXM is detectable in CSF and
organisms. The cryptococcal antigen test is performed by latex agglutination or enzyme
serum
immunoassay, and its quantitation has prognostic significance. A rising antigen level can
indicate progression, and a declining titer is a favorable sign.

FIGURE 47–2. Cryptococcal


meningitis. The C neoformans cells
are stained red by this PAS (periodic
acid-Schiff ) stain. The capsule is
not stained but is creating the halo
around the organisms. Note the lack of
inflammatory cells. (Reproduced with
permission from Connor DH, Chandler
FW, Schwartz DQ, et al: Pathology
of Infectious Diseases. Stamford, CT:
Appleton & Lange, 1997.)
CHAPTER 47 T H E S Y S T E M I C F U N G A L PAT H O G E N S 791791

TREATMENT
The primary treatment for serious cryptococcal infections includes induction therapy with
Amphotericin B plus flucytosine, followed by an extended consolidation course of flucon-
azole. Azoles alone can often be used in non-CNS disease; therefore, it is very important
to perform lumbar punctures and CSF analysis in all patients with cryptococcal infection Amphotericin and flucytosine
to determine if the CNS is involved. Although 75% of persons with meningitis respond to used in combination
this treatment, many patients suffer relapses after antifungal therapy is stopped, requiring
repeated courses of therapy. In patients with AIDS-associated cryptococcosis, reconstitu- ✺ Fluconazole for non-CNS
tion of the immune system with antiretroviral therapy is very important to prevent recur- infections
rent infection. One half of patients with a microbiological cure have some kind of residual
neurologic damage.

HISTOPLASMA

Overview
Histoplasma capsulatum is a dimorphic fungus the mold form of which is restricted to
certain geographic regions. Histoplasmosis is usually limited to specific endemic areas
where the fungus can be readily isolated from the soil. Patients are infected with this
fungus from environmental sources, and they are usually asymptomatic or experience
a self-limited illness characterized by fever and cough. If affected persons are seen by a
physician, a pulmonary infiltrate and hilar adenopathy may or may not be evident on a
radiograph, complicating the initial diagnosis. Progressive infections show extension in the
lung or enlargement of lymph nodes, liver, and spleen.

HISTOPLASMA CAPSULATUM

Histoplasma capsulatum is one example of a thermally dimorphic fungus (Figure 47–3B),


microorganisms that change growth form depending on the ambient temperature. Com-
mon features of this group of fungi include the fact that most are restricted to particular
geographic locations (regions of endemicity). Most of these fungi enter patients through

Microconidium

Macrophage

Macroconidium

ts Budding yeast

Mycelium CW

Vacuole

A B
FIGURE 47–3. Histoplasma capsulatum. A. Mold phase with hyphae, microconidia, and
tuberculate macroconidia. B. A yeast cell is multiplying (note budding) within a macrophage
phagocytic vacuole. (Reproduced with permission from Willey JM: Prescott, Harley, & Klein’s
Microbiology, 7th edition. McGraw-Hill, 2008.)
792 PART IV PAT H O G E N I C F U N G I

the lung where they establish initial infection. Immune competent patients typically spon-
taneously resolve infections due to these fungi, but the infections can rarely become chronic
or disseminated.
Histoplasma capsulatum grows in a round, yeast-like phase in tissue and in cultures incu-
bated at 37°C. However, when incubated at lower temperatures, such as those most com-
monly encountered in the environment, Histoplasma species grow as a filamentous mold
where it is a saprophyte in soil.

HISTOPLASMOSIS

EPIDEMIOLOGY
The organism is particularly prevalent in certain temperate, subtropical, and tropical zones,
and endemic areas are present in all continents of the world except Antarctica. The largest
and best defined is the US region drained by the Ohio and Mississippi rivers (Figure 47–4).
Microconidia are infectious
More than 50% of the residents of states in this area show radiologic evidence of previous
infection. In some locales, up to 90% demonstrate delayed-type hypersensitivity to
Mold grows in humid soil with
Histoplasma antigens, suggesting that they have been infected at some point in the past.
bird droppings
Point source outbreaks of histoplasmosis have occurred after the inhalational of large
amounts of fungi following disturbances of bird roosts, bat caves, and soil at construction
✺ High prevalence in central
sites. Persons in endemic areas whose employment (agriculture, construction) or avocation
United States
(spelunkers) brings them in contact with aerosolized microconidia are at increased risk.
The infection is not transmitted from person to person. Disease is more common in men,
but there are no racial or ethnic differences in susceptibility.

PATHOGENESIS
Once infection is established, H capsulatum cells live primarily in the lymph nodes, spleen,
bone marrow, and other elements of the reticuloendothelial system. Therefore, this is an
example of a microorganism that has adapted to intracellular growth within phagocytic
✺ Reticuloendothelial system is macrophages. Other examples include M tuberculosis and C neoformans. Like tuberculosis
focus of infection and cryptococcosis, the initial infection with H capsulatum occurs in the lungs after inhala-
tion of infectious conidia. These fungal spores convert to the yeast form after germinating
in the host.
The initial pulmonary infection of histoplasmosis generally spreads to regional lymph
nodes, resulting in a primary focus of paired lung/lymphatic lesions similar to the Ghon

Blastomycosis
Histoplasmosis
Coccidioidomycosis

FIGURE 47–4. Geographic distribution


of systemic fungal infections in the
United States.
CHAPTER 47 T H E S Y S T E M I C F U N G A L PAT H O G E N S 793793

FIGURE 47–5. Histoplasma


capsulatum. This peripheral blood
smear shows two monocytes with
multiple organisms stuffed within their
cytoplasm. Note the size of the yeast
cells, which is very small for fungi.
(Reproduced with permission from
Connor DH, Chandler FW, Schwartz DQ,
et al: Pathology of Infectious Diseases.
Stamford, CT: Appleton & Lange, 1997.)

complex of tuberculosis. Also like tuberculosis, most cases never advance beyond the pri-
mary stage, leaving only a calcified node and pulmonary calcifications as evidence of infec- ✺ Lymphatic spread and
tion. As in tuberculosis, viable fungal cells may remain in these old lesions and reactivate reactivation are similar to
later, particularly if the person becomes immunocompromised. The extent of systemic tuberculosis
spread of H capsulatum within macrophages to other body sites during primary infection
is unknown, but it is likely highly variable and dependent on host-specific immune factors.
Pathologically, histoplasmosis is characterized by granulomatous inflammation with
associated necrosis. Even with special fungal stains, H capsulatum may be difficult to detect
within these infected foci, making a precise pathological diagnosis challenging. Extrapul-
monary spread of histoplasmosis occurs primarily in immunocompromised patients, pri-
marily involving the reticuloendothelial system with resulting enlargement of the liver and ✺ Granulomatous response seen
spleen. In patients with compromised immunity, numerous organisms within macrophages in liver, spleen, and bone marrow
may be found in these organs, in lymph nodes, bone marrow, or even peripheral blood
(Figure 47–5). Populations at particular risk for developing disseminated histoplasmosis
include patients with advanced AIDS as well as those being treated with tumor necrosis
factor-alpha inhibitors.

HISTOPLASMOSIS: CLINICAL ASPECTS

MANIFESTATIONS
Most cases of H capsulatum infection in normal hosts are minimally symptomatic, often
causing mild fever and cough for a few days or weeks. Mediastinal lymphadenopathy and
subtle pulmonary infiltrates may be seen on X-rays. More severe cases are most often char-
acterized by chills, malaise, chest pain, and more extensive lung infiltrates, all of which
Most cases are asymptomatic or
usually resolve without specific therapy. However, most people who resolve the primary
with only fever and cough
infection merely have scattered calcified granulomas in their lungs as X-ray evidence of
healed histoplasmosis. Rarely, residual pulmonary nodules due to histoplasmosis may
✺ Progressive pulmonary disease
continue to enlarge over a period of years, causing a differential diagnostic problem with
shows cavities and weight loss
pulmonary neoplasms. Progressive pulmonary disease can mimic pulmonary tuberculo-
sis, with sputum production, night sweats, weight loss, and even the development of lung
cavities. The clinical course of pulmonary histoplasmosis may infrequently be chronic and
relapsing, with symptoms lasting for several months to years.
Disseminated histoplasmosis generally appears as a nonspecific febrile illness in an
immunocompromised patient, with enlargement of reticuloendothelial organs. The CNS,
skin, gastrointestinal tract, and adrenal glands may also be involved. Painless ulcers on ✺ Dissemination involves
mucous membranes are a common finding; however, given their painless nature, these reticuloendothelial organs,
lesions must be actively sought by clinicians in order to help make the diagnosis. The course mucous membranes, and adrenal
of disseminated histoplasmosis is typically chronic, with manifestations that depend on glands
the organs involved. For example, chronic bilateral adrenal failure (Addison disease) may
develop when the adrenal glands are affected.
794 PART IV PAT H O G E N I C F U N G I

DIAGNOSIS
In most forms of pulmonary histoplasmosis, the diagnostic yield of direct examinations or
culture of sputum is low. Because of their small size, the yeast cells are difficult to see in potas-
Blood and bone marrow exami- sium hydroxide (KOH) preparations, and their morphology is not sufficiently distinctive to
nation require special stains be diagnostic. Selective fungal stains such as methenamine silver demonstrate the organism
but may not differentiate it from other yeasts. Hematoxylin and eosin (H&E)-stained
Immunodiffusion and probes tissue or Wright-stained bone marrow often demonstrates the organisms in their intracel-
used with cultures lular location in macrophages (Figure 47–5). Specimens must be examined carefully under
high magnification. Identification of culture isolates requires demonstration of the typical
conidia and dimorphism. Nucleic acid probes have been developed for culture confirmation.
Antibodies can be detected during and after infection, but their diagnostic usefulness in
endemic areas is limited by false-negative results and cross-reactions in patients with blas-
tomycosis. Rising antibody titers are suggestive of dissemination or relapse. The histoplas-
Culture is required for firm min skin test has been useful in the past to document prior exposure, but the reagents are
diagnosis no longer commercially available. In disseminated disease, blood culture or biopsy samples
of a reticuloendothelial organ are the most likely to contain Histoplasma. Of these cultures,
✺ EIA detects circulating antigen bone marrow culture has the highest yield. The diagnosis of disseminated infection has
been greatly aided by the development of a commonly used enzyme immunoassay (EIA)
detecting a Histoplasma polysaccharide antigen. This test can be performed on blood or
urine samples, and it detects more than 90% of cases of disseminated disease.

TREATMENT
Primary infections and localized lung lesions usually resolve without treatment. For mild
disease localized to the lung, a systemic azole such as itraconazole is commonly used. For
more severe or disseminated disease, initial therapy with amphotericin B is often followed
✺ Amphotericin B and by longer-term therapy with itraconazole. Azoles can be effective in an endemic area for
itraconazole prophylaxis of persons with a high risk of disease, including AIDS patients with low CD4
counts and other immunocompromised patients. The echinocandin class of antifungals is
decidedly less effective against the endemic fungal pathogens, and these agents should not
routinely be used for histoplasmosis.

Patients with immunocompromising disorders (eg, patients with progres-


sive AIDS, patients receiving anti-tumor necrosis factor alpha therapy) are
at high risk for developing life-threatening complications from invasive
infections due to pathogenic fungi. Can you suggest minimally invasive
testing strategies that might identify patients with subclinical infections
due to these fungi and who are therefore at risk for serious complications
with progressive immunosuppression?

BLASTOMYCES

Overview
Blastomyces dermatitidis is a dimorphic fungus similar to Histoplama. Many clinical features
of blastomycosis are similar to histoplasmosis. During initial infection, most patients are
asymptomatic or have self-limited mild fever and cough. Chronic infections of the lung
infrequently occur. Skin lesions are the most common manifestation of disseminated
blastomycosis. Unlike histoplasmosis, the reticuloendothelial system is not involved.

Think Apply 47-1. Patients with latestage AIDS or other forms of


immunosuppression can have subclinical infections due to pathogenic fungi that
become clinically evident with time. Minimally invasive tests for fungal antigens, such
as urine or blood tests for Cryptococcus or Histoplasma antigens, have been used
to identify these infected but asymptomatic patients in order to effectively direct
antifungal therapy before serious complications develop.
CHAPTER 47 T H E S Y S T E M I C F U N G A L PAT H O G E N S 795795

FIGURE 47–6. Blastomyces


dermatitidis. Large thick-walled
yeast cells are shown in this sputum.
Note how the blastoconidia retain a
broad attachment to the mother cell
before separating. (Reproduced with
permission from Connor DH, Chandler
FW, Schwartz DQ, et al: Pathology
of Infectious Diseases. Stamford, CT:
Appleton & Lange, 1997.)

BLASTOMYCES DERMATITIDIS

Blastomyces dermatitidis is a dimorphic fungus with some characteristics similar to those of ✺ Blastomyces—Large yeast cells
Histoplasma. Growth develops in the yeast phase in tissues and in cultures incubated at 37°C. have broad-based buds (rule of
The yeast cells are typically larger (8-15 mm) than those of H capsulatum, with broad-based “B’s”)
buds (blastoconidia) and a thick wall (Figure 47–6). The mold phase appears in culture at 25°C.
Mold has small oval conidia
similar to Histoplasma
BLASTOMYCOSIS

EPIDEMIOLOGY
Blastomycosis is most commonly observed in geographic regions that overlap with those for
histoplasmosis. In North America, this includes the upper Midwestern states and regions
✺ Geographic distribution
around the Great Lakes (Figure 47–4), but infections have been reported in Africa, the
similar to Histoplasma
Middle East, and Europe as well. Unlike histoplasmosis, the causative agent of blastomycosis,
B dermatitidis, is not strongly associated with bird or bat habitats.

BLASTOMYCOSIS: CLINICAL ASPECTS

MANIFESTATIONS
Because mild cases of blastomycosis are difficult to diagnose, most infections are only recognized
if they progress to more advanced or disseminated stages of the disease. Pulmonary infection is
evidenced by cough, sputum production, chest pain, and fever. Hilar lymphadenopathy may be ✺ Pulmonary blastomycosis is
present, as may nodular pulmonary infiltrates with alveolar consolidation. This nonspecific clin- similar to other mycoses
ical picture may mimic a pulmonary tumor, tuberculosis, or some other form of chronic pneu-
monitis. Skin lesions are among the most common manifestations of disseminated infection. ✺ Skin lesions are on exposed
In contrast to histoplasmosis, mucous membrane lesions are rarely observed in blastomycosis, surfaces, often as grouped
with lesions developing more commonly on exposed skin often as “grouped microabscesses.” microabscesses
Given the chronicity of many untreated cases of cutaneous blastomycosis, the associated exten-
sive necrosis and fibrosis may produce considerable disfigurement. Bone infection has features ✺ Genitourinary tract/prostate is
similar to those of other causes of chronic osteomyelitis. The urinary and genital tracts are the frequently involved
most commonly affected visceral sites; the prostate is especially prone to infection.

DIAGNOSIS
Direct demonstration of typical large yeasts with broad-based buds (blastoconidia) in KOH
preparations of infected tissue is the most rapid means of diagnosis (Figure 47–6). Biopsy
specimens also have a high yield, and the organisms are visible in histopathology samples
796 PART IV PAT H O G E N I C F U N G I

stained with either H&E or special fungal stains. Blastomyces dermatitidis grows in the
KOH and biopsy show budding clinical microbiology laboratory on routine fungal media, but cultures may take as long
yeast as 4 weeks. Conidia are not particularly distinctive, and demonstration of thermal dimor-
phism and typical yeast morphology is essential to avoid confusion with other fungi. A
Culture takes weeks and conidia DNA probe is particularly useful in differentiating cultures from Histoplasma. Serological
not distinctive tests are available but are less sensitive than those for other fungal pathogens.

TREATMENT
As with histoplasmosis, itraconazole and other mold-active azoles may be used for mild to
Amphotericin B and azoles are moderate disease. Amphotericin B is indicated for more serious or disseminated infections.
effective Fluconazole or voriconazole may be used in meningitis. As with other systemic mycoses,
response to treatment is slow, and relapse is common.

COCCIDIOIDES

Overview
Coccidioides species are dimorphic fungi endemic to parts of the American West. Acute
primary infection with C immitis is most often asymptomatic, but it can manifest as a complex
of symptoms called “Valley Fever” by residents of the endemic areas. Valley Fever includes
fever, malaise, dry cough, joint pains, and sometimes a rash. There are few diagnostic
physical or radiologic findings, but the illness persists for weeks. Disseminated forms of the
disease can involve lesions in the bones, joints, skin, and a progressive chronic meningitis.

COCCIDIOIDES IMMITIS AND COCCIDIOIDES POSADASII

Coccidioides species are dimorphic fungi commonly encountered within North America in
the desert regions of the southwestern United States and northern Mexico. In contrast
Dimorphism involves unique
to Blastomyces and Histoplasma species that grow in the body as budding yeast-like cells,
spherule
the tissue form of Coccidioides species is a large (12-100 μm), round-walled spherule
(Figure 47–7A). This structure is quite distinctive and unique among the pathogenic fungi. Its
✺ Spherules differentiate to form formation takes place in a process illustrated in Figure 47–8. The spherule eventually ruptures,
and release endospores releasing 200 to 300 endospores (Figure 47–9), each of which can differentiate into another
spherule.

A B
20 m
FIGURE 47–7. Coccidioides immitis. A. Lung tissue with a large thick-walled spherule
containing multiple endospores. The smaller spherule to its left has ruptured releasing endospores.
(Reproduced with permission from Connor DH, Chandler FW, Schwartz DQ, et al: Pathology of
Infectious Diseases. Stamford, CT: Appleton & Lange, 1997.) B. Mold phase in which alternate cells
have differentiated to form barrel-shaped arthroconidia. (Reproduced with permission from Nester
EW: Microbiology: A Human Perspective, 6th edition. 2009.)
CHAPTER 47 T H E S Y S T E M I C F U N G A L PAT H O G E N S 797797

Released
endospores

In vivo
cycle
Enlarging
spherule with
cleavage planes

Mature
spherule

Arthroconidia
Hyphae
FIGURE 47–8. Life cycle of
Coccidioides immitis. The nature cycle
Hypha takes place in desert climates with
modest rainfall. Hyphae differentiate
Environment into arthroconidia, which break loose
cycle
Germination and may be suspended in the air. Soil
disruptions and wind facilitate spread
and the probability of inhalation into
human lungs. In the human host
environment, in vivo differentiation
produces cleavage planes and
eventually huge spherules. The
spherules rupture releasing endospores,
which can then repeat the in vivo cycle.

FIGURE 47–9. Coccidioides immitis.


This electron micrograph of infected
mouse lung shows a spherule filled
with endospores (E) and one that has
discharged its endospores into the
surrounding tissue. Note the thickness
of the spherule wall (SW). (Reprinted
with permission from Drutz DJ,
Huppert M. J Infect Dis. 1983;147:379,
Figure 7. Copyright University of
Chicago Publisher.)
798 PART IV PAT H O G E N I C F U N G I

COCCIDIOIDOMYCOSIS

EPIDEMIOLOGY
Coccidioidomycosis is the most geographically restricted of the systemic mycoses because
C immitis grows only in the alkaline soil of semiarid climates known as the Lower Sonoran
life zone (Figure 47–4). These areas are characterized by hot, dry summers, mild winters
✺ Geographically restricted to with few freezes, and annual rainfall of about 10 inches during brief rainy seasons. Ecologic
Sonoran Desert “islands” with these conditions are found scattered throughout Central and South America.
The primary endemic zones in the United States are in Arizona, Nevada, New Mexico, west-
High proportion of locals have ern Texas, and the arid parts of central and southern California. Three unrelated cases in the
been infected eastern half of Washington State could give this zone its most northern extension. The area
between the Cascade and Rocky Mountains is also dry and arid, but prolonged winter freezes
make it less hospitable for Coccidioides species. Persons living in the endemic areas are at
high risk of infection, and positive skin test rates of 50% to 90% occur in long-time residents
of highly endemic areas. Coccidioidomycosis is not transmissible from person to person.
Infection cannot typically be acquired without at least visiting an endemic area, although
some interesting examples have been recorded in which the endemic zone itself “paid a
visit” and resulted in infections. In 1978, a storm originating in Bakersfield, California
(endemic zone), carried a thick cloud of dust all the way to San Francisco. This weather
event was followed by cases of coccidioidomycosis in persons who had never left the Bay
Area. Similarly, infection in a patient who had never left the southeastern United States
was epidemiologically associated with exposure to pre-processed cotton grown in Arizona.
Coccidioides immitis is also a notorious cause of infection in laboratory workers. The high
✺ Considered a potential infectivity of cultured arthroconidia has caused it to be classified as a significant biohazard
bioweapon and potential bioweapon.

IMMUNITY
Lifelong immunity to coccidioidomycosis clearly develops in most of those who become
infected. This immunity is associated with strong polymorphonuclear leukocyte and TH1-
✺ Progressive disease develops
mediated responses to coccidioidal antigens. The disease progresses when cell-mediated
in patients with AIDS or defects
immunity and consequent macrophage activation do not develop. Such immune deficits
in cell-mediated immunity
may be a result of disease (AIDS) or immunosuppressive therapy, but progressive coccidi-
oidomycosis may infrequently occur in persons with no known immune defects.
Humoral mechanisms are not known to play a major role in immunity to coccidioido-
mycosis. In fact, C immitis is resistant to complement-mediated killing, and levels of com-
plement-fixing antibody are inversely related to the process of disease resolution. Persons
Antibody production is inversely with minimal objective indications of tissue involvement (eg, lesions and radiographs)
related to disease progression have strong T-lymphocyte responses to C immitis antigens and little if any detectable anti-
Coccidioides antibody. Those with disseminated disease and absent cellular immunity have
high titers of antibody. Thus, the levels of antibody seem to indicate the degree of antigenic
stimulation rather than any known contribution to resolution of the infection.

COCCIDIOIDOMYCOSIS: CLINICAL ASPECTS

MANIFESTATIONS
More than 50% of those infected with C immitis experience no symptoms, or the disease
is so mild that it cannot be recalled when evidence of infection (serology, skin test) is dis-
covered. Others develop malaise, cough, chest pain, fever, and arthralgia 1 to 3 weeks after
infection. This illness, dubbed Valley Fever by the local San Joaquin Valley residents, lasts 2
to 6 weeks with few distinctive findings. The chest X-ray is usually clear or shows only hilar
CHAPTER 47 T H E S Y S T E M I C F U N G A L PAT H O G E N S 799799

adenopathy. Red, inflamed skin nodules, known as erythema nodosum, may occur during
the course of initial Coccidioides infection, particularly on the extremities. Occurring most ✺ Valley fever usually
frequently in women, these nodules may provide a clinical clue to the cause of infectious self-limiting
symptoms. In most cases, all clinical symptoms of Valley Fever resolve spontaneously, but
often only after considerable discomfort and loss of productivity. In more than 90% of cases, ✺ Erythema nodosum common
there are no pulmonary residua. A small number of cases progress to a chronic pulmonary in women
infection characterized by cavity formation and a slowly relapsing course that extends over
years. Less than 1% of all primary infections and 5% of symptomatic cases disseminate to Chronic and disseminated
foci outside the lung. disease less than 1%
There is a well-recognized but poorly understood predisposition to chronic infections
among distinct patient populations. Disseminated disease is more common in men, as well
as people from sub-Saharan Africa and southeast Asia, particularly the Philippines. Given
the importance of CD4-mediated immunity in controlling coccidioidomycosis, patients ✺ Genetic factors and immune
with AIDS or transplants are also at particular risk for disseminated infection. Evidence of status are predictors for
extrapulmonary infection almost always appears in the first year after infection. The most dissemination
commonly involved sites are bones, joints, skin, and the central nervous system. Coccidi-
oidal meningitis develops slowly with gradually increasing headache, fever, neck stiffness, ✺ Meningitis is chronic
and other signs of meningeal irritation. The CSF findings are similar to those in tubercu-
losis and other fungal causes of meningitis, such as C neoformans. Mononuclear cells pre-
dominate in the cell count, but substantial numbers of neutrophils and eosinophils are often
present. If untreated, the disease is slowly progressive and fatal.

DIAGNOSIS
With enough persistence, direct examination of infected tissue can reveal diagnostic forms
of C immitis. The thick-walled spherules are so large and characteristic (Figure 47–7A)
that they are difficult to miss in wet mounts (KOH, calcofluor) or biopsy sections. Skin Direct examination for spherules
and visceral lesions are most likely to demonstrate spherules; however, these fungal forms can be diagnostic
are rarely seen in the CSF. Spherules released into expectorated sputum are often small
(10-15 mm) and immature without well-developed endospores, thus difficult to visualize.
In contrast, spherules stain well in histologic sections of infected tissue using either H&E
or special fungal stains.
Culture of C immitis from sputum, visceral lesions, or skin lesions is not difficult, but Culture from CSF may be difficult
must be undertaken only by those with experience and proper biohazard protection. Cul-
tures of CSF are positive in less than half the cases of meningitis. Laboratories must be Substantial risk of laboratory
warned of the possibility of coccidioidomycosis to ensure diagnosis and prevent inadver- infection with arthroconidia
tent laboratory infection.
Serologic tests are particularly useful in the diagnosis and management of coccidioi-
domycosis (Figure 47–10). One half to three-quarters of patients with primary infection
develop serum anti-Coccidioides IgM antibody during the first 3 weeks of illness. IgG anti-
bodies (measured by complement fixation) appear in the third week or later, and their titer

Coccidioidomycosis
128

64
e
as
Antibody titer

32
dise
te d
16 ina Treatment/recovery
s em
IgG Dis
8 IgM

2
Valley fever

1 2 3 4 1 2 3 4 5 6 >6
FIGURE 47–10. Serologic tests in
Weeks Months coccidioidomycosis.
800 PART IV PAT H O G E N I C F U N G I

and duration depend on the extent of disease. IgG disappears with resolution or successful
Coccidioidin skin test remains treatment, and it persists with continuing infection. In an appropriate clinical setting, the
positive for life detection of specific antibodies can confirm the diagnosis of coccidioidomycosis, but their
absence does not exclude it. In managing disseminated disease, the magnitude of the IgG
Precipitating IgM indicates acute titer is a measure of the extent of disease, and the direction of any change is often an indica-
infection tion of prognosis. For example, a high (>1:32) and rising titer indicates a poor prognosis.
The presence of IgG in the CSF is also important in the diagnosis of coccidioidal meningitis
✺ IgG detected by complement because cultures are frequently negative. The coccidioidin skin test was also a useful tool to
fixation quantitates disease indicate prior exposure, but it is no longer routinely available.

TREATMENT
Primary coccidioidomycosis is self-limited, and no antifungal therapy is indicated except
Primary disease treated only with to reduce the risk of dissemination in patients with risk factors, such as immunocompro-
risk factors mise and pregnancy. Itraconazole is preferred for acute or progressive pulmonary disease,
with fluconazole as an alternative agent. Disseminated, extrapulmonary infection may
Amphotericin B and azoles in require amphotericin B. Fluconazole is often favored as treatment for meningitis because
progressive disease of its enhanced CSF penetration. In cases of refractory meningitis, amphotericin B may be
infused directly into the CSF. Unlike other forms of the disease that can be treated for cure,
coccidioidomycosis involving the CNS often requires lifelong therapy.

PARACOCCIDIOIDES BRASILIENSIS
Paracoccidioides brasiliensis is the cause of paracoccidioidomycosis (South American
blastomycosis), a disease limited to tropical and subtropical areas of Central and South
America. The organism is a dimorphic fungus, the most noteworthy feature of which is
the production of multiple blastoconidia from the same cell. Characteristic 5 to 40 μm
Yeast with multiple blastoconidia cells covered with budding blastoconidia may be seen in tissue or in yeast-phase growth at
are seen in ulcerative lesions 37°C. This structure has a morphology reminiscent of a ship’s steering wheel, often referred
to as a “Captain’s wheel” when seen in tissue. The disease manifests primarily as chronic
mucocutaneous or cutaneous ulcers. The disfiguring ulcers spread slowly and develop a
granulomatous mulberry-like base. Regional lymph nodes, reticuloendothelial organs, and
the lungs may also be involved.
Paracoccidioidomycosis has a striking predilection for men, despite skin test evidence
that subclinical cases occur at the same rate in both sexes. This may be related to the exper-
Disease has a strong predilection imental observation that estrogens but not androgens inhibit conversion of mold-phase
for men conidia to the yeast phase. Treatment is with sulfonamides, amphotericin B, and, more
recently, the azole compounds.

KEY CONCLUSIONS
• Remember the “C’s” of Cryptococcus neoformans
• Capsule
• CD4 cell dysfunction predisposes to symptomatic infection
• CNS infections are most common
• Cryptococcal antigen is the most sensitive diagnostic test
• The thermally dimorphic fungi each have distinctive tissue forms
• Histoplasma—small, budding yeasts
• Blastomyces—broad-based budding yeasts
• Coccidioides—spherule
• Histoplasmosis can mimic tuberculosis with a primary lung infection,
granulomatous inflammation, and dissemination in macrophages.
• Blastomycosis also begins with a lung infection but disseminated disease often
manifests as chronic skin lesions.
• Coccidioidomycosis can cause a chronic and fatal meningitis.
CHAPTER 47 T H E S Y S T E M I C F U N G A L PAT H O G E N S 801801

CASE STUDY
A FORGETFUL FARMER
A 64-year-old white male farmer was hospitalized because of “progressive
dementia.” He had been in excellent health and working full time, never having
left the state of Montana. Eight months before admission, he became sloppy,
careless, forgetful, and at times confused. These symptoms remitted somewhat,
and he was able to perform his work on the farm. His family felt he was normal
except for mild impairment of recent memory. One month before admission
the symptoms recurred, and he complained of headache. After this, he became
progressively worse and was finally brought to the hospital.
Physical examination revealed a well-developed man who did not appear ill.
His blood pressure, pulse and respiratory rate were normal, and his temperature
was 99.2°F. The rest of the examination was normal except for mild nuchal rigidity,
disorientation to time and place, and marked confusion. A complete blood count
was normal, and an HIV serological test was negative.
Lumbar puncture revealed clear CSF with an opening pressure of 250 mm; 100
white blood cells, all of which were mononuclear; protein of 85 mg% and glucose of
45 mg% (concomitant blood sugar was 90 mg%). Gram stain of the CSF was negative.

QUESTIONS
1. If this is a case of fungal meningitis, the most likely etiologic agent is:
A. Candida albicans
B. Cryptococcus neoformans
C. Histoplasma capsulatum
D. Coccidioides immitis
E. Paracoccidioides brasiliensis
2. If blood and CSF cultures for bacteria, mycobacteria, and fungi are negative,
what test is most likely to reveal the diagnosis?
A. GXM antigen detection
B. GXM antibody detection
C. Germ tube test
D. Silver stain
E. Coccidioides immitis IgG
3. What is the most common route of primary infection for this pathogenic
microorganism?
A. Inoculation
B. Ingestion
C. Insect vector
D. Inhalation
E. Animal bite

ANSWERS
1. (B), 2. (A), 3. (D)
Most cases of cryptococcal meningitis/meningoencephalitis occur in patients
with defined defects of immunity, such as late-stage HIV infection. However,
this infection has been described in otherwise immunocompetent patients,

You might also like